29
MINISTRY OF PUBLIC HEALTH OF UKRAINE Department of human resources policy, education and science Testing Board Test items for licensing examination Krok 2 STOMATOLOGY Student ID Last name Variant ________________

Krok 2 STOMATOLOGY - Номеmmf.dsmu.edu.ua/attachments/article/24/E94t10_2016E.pdf · 2018-06-06 · Krok 2 Stomatology (англомовний варiант, iноземнi студенти)

  • Upload
    others

  • View
    46

  • Download
    2

Embed Size (px)

Citation preview

Page 1: Krok 2 STOMATOLOGY - Номеmmf.dsmu.edu.ua/attachments/article/24/E94t10_2016E.pdf · 2018-06-06 · Krok 2 Stomatology (англомовний варiант, iноземнi студенти)

MINISTRY OF PUBLIC HEALTH OF UKRAINE

Department of human resources policy, education and science

Testing Board

Test items for licensing examination

Krok 2

STOMATOLOGY

Student ID Last name

Variant ________________

Page 2: Krok 2 STOMATOLOGY - Номеmmf.dsmu.edu.ua/attachments/article/24/E94t10_2016E.pdf · 2018-06-06 · Krok 2 Stomatology (англомовний варiант, iноземнi студенти)

ББК 54.1я73

УДК 61

Authors of items: Amosova L.I., Andrianova I.I., Avdieiev O.V., Babenko L.M.,

Babiychuk N.F., Bas O.A., Bedeniuk O.A., Bielikov O.B, Bondalietov V.O., Bondarenko V.S.,

Bosa L.F., Chaikivsky R.V., Cherepynska Yu.A., Chernov D.V., Chukhrai N.L.,

Chumachenko V.A., Chyhrynets V.N., Chyzhevsky I.V., Demkovych A.Ye., Derebaliuk L.Ya.,

Derkach L.Z., Devdera O.I., Dmytrenko R.R., Dmytriieva A.A., Dubrovina O.V., Duda K.M.,

Dzetsiukh T.I., Fastovets O.O., Havaleshko V.P., Halahdyna A.A., Han I.V., Hlushak A.A.,

Hodovanets O.I., Holik V.P., Holovko N.V., Holubieva I.M., Hrechko N.B., Hrekuliak V.V.,

Idashkina N.H., Isakova N.M., Ivchenko N.A., Karelina L.S., Kaskova L.F., Kharkov L.V.,

Khomych N.M., Kindii V.D., Konovalov M.F., Kosarieva L.I., Koval S.M., Kovalchuk L.P.,

Kril A.Y., Krupnyk N.M., Kryklias H.H., Kudinov V.O., Kuz V.S., Kuz H.M., Kuzniak N.B.,

Kyryliuk M.I., Larionov I.M., Lavrovska O.M., Levko V.P., Lokes K.P., Lomeks O.I.,

Lysenko Yu.H., Makarevych A.Yu., Mazurina I.O., Mikhalova A.O., Moiseitseva L.O.,

Muntian O.V., Muzychina H.A., Neiko N.V., Nesyn O.F., Novytska I.K., Odzhubeiska O.D.,

Oleniychuk V.V., Oliynyk O.V., Orlovsky V.O., Ovcharenko E.N., Ozhohan Z.R., Pankevych A.I.,

Pantus A.V., Parasochkina V.V., Pasko O.O., Pavelko N.M., Petrushanko T.O., Piuryk V.P.,

Plekhova M.M., Plotnikova V.H., Potiyko V.I., Prodanchuk A.I., Prots H.B., Pushkova T.M.,

Rekova L.P., Rimsha O.Ye., Riznyk S.S., Romanenko O.H., Romanovska A.P., Rubizova A.H.,

Ruda I.V., Ruzin H.P., Samsonov O.V., Semenenko I.P., Senchenko O.M., Shakhnovsky I.V.,

Sharan M.O., Shcherbyna I.M., Shubladze H.K., Shuklin V.A., Shutak O.V., Shuvalov S.M.,

Skakun L.M., Smahliuk L.V., Spichka I.A., Stakhanska O.O., Steblianko L.V., Svirchkov V.N.,

Sydorova A.I., Tyshchenko V.I., Tsentilo V.H., Udod O.O., Usenko S.A., Valchuk O.H.,

Vasylenko V.M., Voropaieva L.V., Yatsenko I.V., Zablotsky Ya.V., Zaitsev L.O., Zavoiko L.M.,

Zelinska N.A., Zhero N.I.. and Committees of professional expertise.

Item reviewers. Bezvushko E.V., Bulbyk O.I., Chyzhevsky I.V., Dmytriieva A.A., Fastovets O.O.,

Gerelyuk V.I., Ilenko N.M., Kaskova L.F., Lungu V.I., Muntian L.M., Novikov V.M., Ostapko O.I.,

Smagliuk L.V., Solovey S.I., Tril S.I., Tsentylo V.G., Volynets V.M., Volyak M.N.

The book includes test items for use at licensing integrated examination “Krok 2. Stomatology” and

further use in teaching.

The book has been developed for students of stomatological faculties and academic staff of higher

medical educational establishments.

Approved by Ministry of Public Health of Ukraine as examination and teaching

publication based on expert conclusions (Orders of MPH of Ukraine of

14.08.1998 №251, of 27.12.1999 №303, of 16.10.2002 №374, of 29.05.2003 №233).

© Copyright Testing Board.

General Instruction

Every one of these numbered questions or unfinished statements in

this chapter corresponds to answers or statements endings. Choose the

answer (finished statements) that fits best and fill in the circle with the

corresponding Latin letter on the answer sheet.

Page 3: Krok 2 STOMATOLOGY - Номеmmf.dsmu.edu.ua/attachments/article/24/E94t10_2016E.pdf · 2018-06-06 · Krok 2 Stomatology (англомовний варiант, iноземнi студенти)

Krok 2 Stomatology (англомовний варiант, iноземнi студенти) 2016 рiк 1

1. A prosthodontist plans to make aporcelain-fused-to-metal crown for the 23tooth. In order to correct its abnormal posi-tion in the dental arch the prosthododntistis going to use a cast post and core. What isthe maximum permissible value (in degrees)of the core deviation from the tooth axis?

A. 15B. 10C. 6-8D. 4E. 20-25

2. What impression material is used toobtain impressions for making orthodonticappliances?

A. YpeenB. StomaflexC. RepinD. SielastE. Orthocor

3. A 45-year-old woman consults a denti-st for prosthetics. Objectively: the 47, 46,45, 35, 36, 37 teeth are missing. There isdentoalveolar vertical displacement of the17, 16, 26, 27 teeth; the alveolar process ishypertrophied, the cervical margins are notexposed. When the teeth are closed, the di-stance between the occlusal surfaces of theteeth and alveolar process is approximately6 mm. Specify the most efficient method oftreatment of secondary deformation in thispatient:

A. Pulp removal and teeth grindingB. Surgical treatmentC. GrindingD. Hardware-associated surgical treatmentE. Disocclusion

4. A 35-year-old patient has made an appoi-ntment with a doctor to have oral cavitysanation. Objectively: the vestibular surfaceof the 22 tooth has a hard tissue defectlocalized in the mantle dentin; the dentin ishard, dark-coloured, the floor of the cavityis coarse. Cold stimulus and probing are pai-nless. What is the most probable diagnosis?

A. Chronic median cariesB. Enamel erosionC. Cuneiform defectD. Dental fluorosis, destructive formE. Hard tissue necrosis

5. A 25-year-old female patient consulteda dentist about acute pain in the mandi-ble on the right that occurs during eating.Objectively: the approximate distal surfaceof the 45 tooth exhibits a carious cavity filledwith light softened dentin. Probing causesa minor pain response across the entirefloor. Percussion causes no pain. Cold water

causes transient pain. What is the most li-kely diagnosis?

A. Acute deep cariesB. Acute median cariesC. Chronic deep cariesD. Chronic median cariesE. Chronic fibrous pulpitis

6. A 42-year-old patient complains of pain inthe submaxillary and sublingual areas thataggravates during eating, body temperaturerise up to 37, 6oC. He has been sufferingfrom this for 2 months. Objectively: infi-ltration along the right sublingual torus,hyperemia, soft tissue edema, acute painduring palpation. The duct of the rightsubmandubular salivary gland excretesturbid saliva mixed with pus. What is themost likely diagnosis?

A. Exacerbation of salivolithiasisB. Acute purulent lymphadenitis of submaxi-llary areaC. Adenophlegmon of submaxillary areaD. Abscess of maxillolingual grooveE. Retention cyst of sublingual salivary gland

7. A patient consulted a dental surgeonabout fever up to 37, 6oC, soft tissue swelli-ng, pain in the 47 tooth on the lower jaw onthe left, the pain aggravates when the toothis touched by its antagonist. Objectively:mucogingival fold of the alveolar processis hyperemic and flat on the vestibular sideof the decayed 47 tooth. What is the mostlikely diagnosis?

A. Acute purulent odontogenic periostitisB. Acute serous periodontitisC. Exacerbation of chronic periodontitisD. Acute odontogenic osteomyelitisE. Periodontal abscess

8. A 12-year-old boy complains of a cavityin the tooth on the lower left jaw. Objecti-vely: 1/3 of the 36 tooth is destroyed, thecarious cavity opens into the dental cavity;reaction to cold stimulus is positive; probingand percussion are painless. X-ray imagi-ng shows the periodontal fissure of the 36thtooth roots to be widened. What is the provi-sional diagnosis?

A. Chronic fibrous periodontitisB. Chronic simple pulpitisC. Chronic gangrenous pulpitisD. Chronic granulating periodontitisE. Chronic granulomatous periodontitis

9. A 5-year-old child has been diagnosed wi-th chronic granulating periodontitis of the85 tooth. What is the optimal material forthe root canal filling?

Page 4: Krok 2 STOMATOLOGY - Номеmmf.dsmu.edu.ua/attachments/article/24/E94t10_2016E.pdf · 2018-06-06 · Krok 2 Stomatology (англомовний варiант, iноземнi студенти)

Krok 2 Stomatology (англомовний варiант, iноземнi студенти) 2016 рiк 2

A. Zinc-eugenol cementB. Zinc-phosphate cementC. Glass ionomer cementD. Calcium-containing pasteE. Resorcinol-formalin paste

10. A patient complaining of constant dullpain in the 38 tooth has made an appoi-ntment with a dentist. Mouth opening isrestricted to 1 cm. The patient is prescri-bed extraction of the 38 tooth. What kind ofanesthesia should be administered?

A. Веrcher-Dubov anaesthesiaB. MandibularC. TorusalD. Extraoral administration of mandibularanaesthesiaE. Tuberal

11. A 67-year-old patient consulted anorthodontist about missing of the 34, 35,36, 37, 45, 46 teeth. Orthopantomogramof the remaining teeth shows the alveolarbone resorption in the range of 1/3 of theinterdental septa height. What constructionshould be offered to the patient?

A. Clasp prosthesis with splinting elementsB. Bridges on the posterior teethC. Removable partial lamellar dentureD. Cap splintE. Van Thiel splint

12. A 22-year-old student complains of gi-ngival hemorrhage during tooth brushi-ng. Objectively: hyperemia and edema ofmarginal gingiva of the front teeth on bothjaws, periodontal pockets are absent. X-rayreveals no pathological changes. What is themost likely diagnosis?

A. Chronic catarrhal gingivitisB. Localized periodontitisC. Ulcerative gingivitisD. Hypertrophic gingivitisE. Generalized periodontitis

13. A 32-year-old male patient complainsof dryness and burning pain in the back ofthe tongue. The symptoms have been lastingfor 1 week. The pain increases when spicyfood is taken. A day prior to that the patienthad suffered from pneumonia and had beentaking antibiotics. Objectively: the skin andmucosa of oral cavity are pale. The tonguemucosa is hyperemic and swollen; thereis crumbling grey-and-white coating in thefolds of the back of the tongue; the lateralsurfaces of the tongue are desquamated.The saliva is thick and drags in threads aftera spatula. Choose the most effective drugcomplex for topical treatment:

A. Methylene blue + Pimafucin (Natamycin)B. Hydrocortisone ointment + SolcoserylC. Decaminum (Dequalinium chloride) +HydrocortisoneD. Furacilinum (Nitrofural) + MethyleneblueE. Furacilinum (Nitrofural) + Solcoseryl

14. A 46-year-old patient complains ofspontaneous pain in the 36 tooth. Its crownis decayed. The patient presented withedema of soft tissues adjacent to the mandi-ble. Body temperature rose up to 39, 0oC,the patient has chill. Examination of theoral cavity revealed a pronounced edemaof the left mandibular alveolar process onthe vestibular and oral side (acute periosti-tis); the 35, 36, 37 teeth are mobile, subgi-ngival pockets contain purulent exudate;their percussion causes acute pain. Positi-ve Vincent’s symptom is present. What isthe presumptive diagnosis?

A. Acute odontogenic osteomyelitis ofmandible from the 36 toothB. Exacerbation of chronic periodontitisfrom the 36 toothC. Acute purulent periostitis of mandiblefrom the 36 toothD. Exacerbation of generalized periodontitisE. Exacerbation of chronic mandibularosteomyelitis

15. An 18-year-old student needs prostheticporcelain-fused-to-metal denture for the11, 21 teeth. There are no contraindicati-ons for the use of such construction. Whatis the most appropriate material for takingimpressions?

A. SielastB. StomalginC. OrthocorD. StensE. Repin

16. A 20-year-old patient addressed a dentistfor treatment of the destroyed teeth. Exami-nation revealed round painless papulae wi-th hyperemic infiltrated crown 7 mm in di-ameter on the palatine arches, soft palate,tonsils, tongue. Regional lymph nodes areenlarged, dense, elastic, painless and mobi-le. What tactics should a doctor choose?

A. Refer for a consultation to a venereologistB. Perform bacteriological analysisC. Refer for a consultation to an ENT doctorD. Perform oral cavity sanationE. Perform cytological analysis

17. A week ago an 18-year-old girl complai-ned of pain in the 22 tooth which was treatedand filled several years ago. Over the pasttwo days the pain increased. Objectively:the 22 tooth is filled, percussion is pai-

Page 5: Krok 2 STOMATOLOGY - Номеmmf.dsmu.edu.ua/attachments/article/24/E94t10_2016E.pdf · 2018-06-06 · Krok 2 Stomatology (англомовний варiант, iноземнi студенти)

Krok 2 Stomatology (англомовний варiант, iноземнi студенти) 2016 рiк 3

nful, mucous membrane is hyperaemic andedematic. Spot-film roentgenograph of the22 tooth shows an ill-defined periapical bonerarefaction 0,4x0,5 cm large. What is themost likely diagnosis?

A. Exacerbation of chronic periodontitis ofthe 22 toothB. Suppuration of the radicular cystC. Acute odontogenic osteomyelitisD. Acute purulent periodontitis of the 22toothE. Acute maxillary periostitis

18. A 22-year-old patient complains of a pai-nful swelling in the right parotid gland. Aweek earlier the patient received a cheekabrasion that healed under the purulentcrust. Over the past two days the patienthad observed progressing pain and fever upto 38, 6oC. Objectively: there is a soft tissueedema in the right parotid region, the skinis slightly strained but has not changed incolour. There is a dense painful infiltrati-on 2,5x3,5 cm large, the skin over it exhibi-ts limited mobility. The mouth can be fullyopened, the mucous membrane around theorifice of the salivary duct is unchanged, thesaliva is transparent. What is the most likelydiagnosis?

A. Acute lymphadenitisB. Exacerbation of chronic parotitisC. Abscess of the parotid-masseteric regionD. Acute non-epidemic parotitisE. Epidemic parotitis

19. A 3-year-old girl complains of pain andswelling in the region of the decayed 51,52 teeth, body temperature rise up to 37,5-37,9oC. Objectively: the face is asymmetricbecause of a swelling in the upper lip regionand right infraorbital region. The crown ofthe 51 tooth is completely decayed. Mucousmembrane in the region of the 52, 51, 1 teethis edematic, mucogingival fold is smoothed,palpation provokes pain, mobility of I-IIgrade of the 51, 52 teeth is also present.What is the most likely diagnosis?

A. Acute purulent odontogenic maxillaryperiostitisB. Acute albuminous odontogenic maxillaryperiostitisC. Acute odontogenic maxillary osteomyeli-tisD. Odontogenic abscess of infraorbital regionE. Exacerbation of chronic periodontitis ofthe 51 tooth

20. A 50-year-old patient has median lowerjaw fracture with formation of a false joint.The 38, 32, 31, 41, 42, 48 teeth are missing.The remaining teeth are intact, stable. Thereis no displacement of lower jaw fragments.X-ray picture shows a bone tissue defect up

to 1 cm large. What prosthesis is indicated?

A. Oxman’s bridge-like prosthesis with pivotpointB. Clasp dentureC. Lamellar prosthesis with Gavrilow’s pivotpointD. Lamellar prosthesis with Oxman’s pivotpointE. Bridge-like prosthesis without a pivotpoint

21. A 32-year-old patient has made an appoi-ntment with a dental surgeon to have oralcavity sanation performed prior to dentureinstallation. During examination the doctorstated the following: the crown of the 25tooth is destroyed. The tooth root is stable,and its percussion is painless. Mucosa of thealveolar process is unchanged. X-ray revealsslight widening of the periodontal fissure.What provisional diagnosis can be made?

A. Chronic fibrous periodontitis of the 25thtoothB. Chronic periodontitis of the 25th toothC. Chronic granulomatous periodontitis ofthe 25th toothD. Chronic granulating periodontitis of the25th toothE. Cystogranuloma

22. A 45-year-old man complains about li-quid outpouring from his nose, inability toblow his nose, inflated cheeks. Objectively:there is a perforating defect (1х1,5 cm) ofalveolar process at a level of the extracted26th tooth in the lateral part of his upperjaw. Air inhalation through the nose withheld nostrils is accompanied by generati-on of bubbles in the area of perforati-on. What denture construction should berecommended?

A. Minor saddle denture with clasp fixationB. Clasp denture with obturating partC. Common partial removable dentureD. Common dental bridgeE. Protective palatal bars

23. A 25-year-old patient presents withfluorosis of vestibular surfaces of the upperincisors. Which of the following constructi-ons will provide the maximum aestheticresult?

A. VeneersB. Plastic crownC. Ceramic crownD. Combined Kurylenko crownE. Metal-plastic crown

24. A 6-year-old child complains of painin the mandibular tooth on the left duringeating. Objectively: masticatory surface ofthe 36 tooth exhibits a carious cavity within

Page 6: Krok 2 STOMATOLOGY - Номеmmf.dsmu.edu.ua/attachments/article/24/E94t10_2016E.pdf · 2018-06-06 · Krok 2 Stomatology (англомовний варiант, iноземнi студенти)

Krok 2 Stomatology (англомовний варiант, iноземнi студенти) 2016 рiк 4

the mantle dentin. The cavity is full of lightsoftened dentin which can be easily removedwith an excavator. Probing of the cavity wallscauses pain response. The teeth are sensitiveto thermal stimuli, the pain is of short-termnature. There is no response to percussion.Select the optimal filling material:

A. Silver amalgamB. Zinc phosphate cementC. Silicate cementD. Silicophosphate cementE. Polycarboxylate cement

25. A 48-year-old patient complains ofsubfebrile temperature and a growing ulceron the gingival mucosa around the molars;teeth mobility in the affected area, cough.Objectively: gingival mucosa in the regionof the lower left molars has two superfici-al extremely painful ulcers with underminededges. The ulcers floor is yellowish, granular,covered with yellowish and isolated pinkgranulations. The ulcers are surrounded bythe tubercles. Dental cervices are exposed,there is a pathological tooth mobility. Regi-onal lymph nodes are enlarged and makedense matted together groups. What is themost likely diagnosis?

A. TuberculosisB. SyphilisC. Acute aphthous stomatitisD. Infectious mononucleosisE. Decubital ulcer

26. A 78-year-old patient is completelyedentulous. He has been wearing denturesfor 19 years. The patient complains of poorfixation of the upper denture. Objectively:the lower third of the face is shortened, thealveolar processes of both jaws are markedlyatrophied, the palate is flat. Mucousmembrane in the denture-supporting areais atrophied. How often should the denturesbe remodelled or restored?

A. Every 3-4 yearsB. Every 6 monthsC. Once a yearD. Every 7 yearsE. Every 10-12 years

27. A 56-year-old man complains of painin the left parotidomasticatory region,progressing face asymmetry that was noticeda month ago. Objectively: left-sided paresisof mimic muscles. To the fore of earflapthere is an ill-defined infiltration, the skinabove it is tense and cyanotic; left lymphnodes are enlarged. Opening of mouth is li-mited down to 2,5 cm. The left parotid ductdoesn’t excrete saliva. What is the most li-kely diagnosis?

A. AdenocarcinomaB. Mixed tumourC. Cyst of the glandD. Glandular tuberculosisE. Chronic lymphadenitis

28. When a prosthodontist was preparingthe patient’s tooth, the patient had epilepticseizure that was subsequently terminated.What mistake had been made by the doctor?

A. No inquire into the patient anamnesisB. No inquire into the antecedent anamnesisC. No anesthesiaD. Crude preparationE. Did not decline the appointment

29. A 13-year-old child complains of havinga cavity in the front maxillary teeth. Contactmedial surfaces of the 11 and 21 teethexhibit cavities found within the mantledentin and filled with dense pigmenteddentin. Probing of the cavity floor causes nopain response, neither does dental percussi-on. Select the best filling material for thepermanent seals:

A. Resin compositeB. Silicate cementC. Silicophosphate cementD. Zinc phosphate cementE. Glass ionomer cement

30. A 30-year-old patient complains oftoothache caused by hot and cold stimuli.The pain irradiates to the ear and temple.Previously there was spontaneous nocturnaltoothache. Objectively: on the occlusalsurface of the 37 tooth there is a deep cari-ous cavity communicating at one point withthe tooth cavity. Probing at the communi-cation point, as well as cold stimulus, causesacute pain. The pain persists for a long time.Electric pulp test result is 55 microamperes.What is the most likely diagnosis?

A. Exacerbation of chronic pulpitisB. Acute diffuse pulpitisC. Exacerbation of chronic periodontitisD. Chronic concrementous pulpitisE. Acute purulent pulpitis

31. An 11-year-old boy complains of a cari-ous cavity in the mandibular tooth on theright. Objectively: the 46 tooth exhibits acarious cavity within the mantle dentin. Thedentin is dense, pigmented; there is painresponse to the cold stimulus; probing andpercussion cause no pain response. Makethe provisional diagnosis:

A. Chronic median cariesB. Acute median cariesC. Acute deep cariesD. Chronic deep cariesE. Chronic superficial caries

Page 7: Krok 2 STOMATOLOGY - Номеmmf.dsmu.edu.ua/attachments/article/24/E94t10_2016E.pdf · 2018-06-06 · Krok 2 Stomatology (англомовний варiант, iноземнi студенти)

Krok 2 Stomatology (англомовний варiант, iноземнi студенти) 2016 рiк 5

32. During preventive examination an 11-year-old girl presents with slight hyperemy,cyanosis, swollen gums, gingival papillaare loose, not adjacent to teeth cervices.Fedorov-Volodkina dental hygiene index is4 points. Schiller’s test is positive. Make thediagnosis:

A. Chronic catarrhal gingivitisB. Hypertrophic gingivitisC. Acute catarrhal gingivitisD. Necrotizing ulcerative gingivitisE. -

33. A 22-year-old medical student complai-ns of the changed colour of his central upperincisor on the right. Two years ago the toothbecame gray. Objectively: the 2 tooth is fi-lled, changed in colour, stable, percussion ispainless. The patient has closed bite. Whatis the absolute contraindication to makingporcelain crowns for this patient?

A. Closed biteB. Defects of the anterior teeth that cannotbe restored with fillingsC. Enamel hypoplasia with a change of toothshape and colourD. Abnormal tooth colourE. Defects of pulpless teeth that cannot berestored with inlays

34. A 25-year-old patient complains of paincaused by eating sweet, hot and cold food;pain ceases, when stimulation stops. Objecti-vely: the adjoining surface of the 36th toothhas a deep carious cavity localised in the ci-rcumpulpar dentin. The dentin is softened.Probing of the carious cavity floor is painful.What is the most probable diagnosis?

A. Acute deep cariesB. Acute median cariesC. Chronic fibrous pulpitisD. Chronic median cariesE. Chronic deep caries

35. A 27-year-old patient complains of thelong-term pain in the 22 tooth caused bycold and hot food, as well as of spontaneouspain lasting for 30 minutes and occurring 3-4 times per day, aggravating at night. Painarose 3 days ago after preparation of thetooth for the acrylic crown. Objectively: the22 tooth is intact, the thermal test causesacute long-lasting pain, percussion is pai-nless. What is the optimal treatment tactics?

A. Vital pulp extirpationB. Application of fluorine lacquerC. Biological treatment of pulpitisD. Devital pulp extirpationE. Vital pulp amputation

36. An 11-year-old boy complains of a short-term pain from the cold in the left mandi-

bular tooth. Objectively: the medial surfaceof the 36 tooth exhibits a carious cavity wi-thin parapulpar dentin. The cavity is filledwith light, softened dentin and does notcommunicate with the cavity of the tooth.Probing the of the 36 tooth floor causespain response, the tooth is not sensitive topercussion, the response to the cold stimulusdoes not remain long after its removal. Whatis the most likely diagnosis?

A. Acute deep cariesB. Acute focal pulpitisC. Acute diffuse pulpitisD. Chronic fibrous pulpitisE. Acute median caries

37. A 46-year-old patient after the recoveryfrom a case of influenza has suddenly startedsuffering from pain in the 36 tooth, thecrown of which had been destroyed. Thesoft tissues surrounding the lower jaw areswollen. There are high fever up to 39oC andchills. Objectively: there is prominent swelli-ng of the left lower jaw alveolar mucosa(both oral and vestibular surfaces). The 34,35 and 37 intact teeth are mobile. Percussi-on is painful. The gingival pockets producepurulent exudate. Vincent’s symptom isobserved. What diagnosis can be suspected?

A. Acute odontogenic mandibularosteomyelitis starting from the 36th toothB. Exacerbation of chronic periodontitis ofthe 36th toothC. Acute suppurative mandibular periostitisstarting from the 36th toothD. PeriodontitisE. Exacerbation of chronic mandibularosteomyelitis

38. A doctor treating an AIDS-positive pati-ent has accidentally pierced his own skin ofthe palm with a root needle. What should bethe first course of actions?

A. Squeeze out blood, process the breachwith 70% alcohol solutionB. Squeeze out blood, process thebreach with strong solution of potassiumpermanganateC. Apply tourniquet to the shoulderD. Process the breach with 5% iodine soluti-onE. Process the breach with 3% hydrogenperoxide solution

39. A 57-year-old patient complains of toothmobility, inability to eat. Objectively: thelower 35, 36, 37, 38, 44, 45, 46 and 48 teethare missing; the 31, 32, 33, 34, 41, 42, 43,47 teeth exhibit II grade mobility, their cli-nical crowns are low, tooth equator is notpronounced. What is the optimal dentureconstruction in this case?

Page 8: Krok 2 STOMATOLOGY - Номеmmf.dsmu.edu.ua/attachments/article/24/E94t10_2016E.pdf · 2018-06-06 · Krok 2 Stomatology (англомовний варiант, iноземнi студенти)

Krok 2 Stomatology (англомовний варiант, iноземнi студенти) 2016 рiк 6

A. Removable cast splintB. Removable partial dentureC. Kurlyandsky splint barD. Removable Bynin splintE. Removable splint with vestibulo-oral clasp

40. A 7-year-old boy complains of increasedbody temperature up to 38oC, headache,sore throat. Objectively: there are erosionson the slightly hyperemic mucosa of the softpalate, anterior pillars of the fauces, tonsils.The submandibular lymph nodes are slightlyenlarged, painless. Name the causative agentof this disease:

A. Coxsackie virusB. Herpes simplex virusC. Epstein-Barr virusD. Klebs-Loeffler bacillusE. Bordet-Gengou bacillus

41. A patient complains of dull ache in the16 tooth, which occurs during eating coldfood. Anamnesis: the tooth had been fi-lled due to deep caries, the filling was lost1 year ago. Objectively: a deep carious cavi-ty that does not penetrate the tooth cavityis present; percussion is painless, probing ispainful along the whole floor of the cariouscavity. Electric pulp test - 50 microamperes.Thermodiagnosis is painful. Make the di-agnosis:

A. Chronic fibrous pulpitisB. Chronic deep cariesC. Chronic fibrous periodontitisD. Acute deep cariesE. Chronic gangrenous pulpitis

42. A patient complains of acute short-termtoothache caused by thermal and chemicalirritants. The tooth has not been treatedbefore, the pain arose about a month ago.Objectively: on the occlusal surface of the36 tooth there is a deep carious cavity wi-thin circumpulpal dentin with overhangingchalky enamel. Probing of the cavity floorcauses slight pain. There is also short-termpain from the cold stimulus. The result ofelectric pulp test is 8 microamperes. What isthe most likely diagnosis?

A. Acute deep cariesB. Acute median cariesC. Pulp hyperemiaD. Chronic deep cariesE. Chronic fibrous pulpitis

43. A 65-year-old patient needs completeremovable dentures for both jaws. At thestage of ”testing the denture constructi-on” the doctor checks the pronunciationof sounds ”S” and ”Z”. Which method ofnormalization of speech functions should beapplied in this case?

A. Phonetic testsB. GraphicC. MyogymnasticsD. SpectrographicE. Acoustic

44. A 7,5-year-old boy fell down on hisface and damaged his front maxillary teeth.Objectively: the crowns of the 11 and 12teeth are destroyed by 1/4. The tooth cavi-ty is closed. Probing causes pain along thebreak-off line, percussion of the 11 and12 teeth causes no pain response. Mucosaaround the 11 and 12 teeth is slightlyhyperemic. Radiograph of the 11 and 12teeth shows intactness of the dental tissuesand alveolar process. What tactics of a denti-st would be optimal in the first hours afterthe injury?

A. Pulp monitoringB. Fabrication of an artificial crownC. Photopolymer fillingD. Parapulpar pin reconstructionE. Composite filling

45. A 56-year-old man complains of painin the left parotid-masticatory area andprogressing facial asymmetry first noticedone month ago. The diagnosis is the cancerof parotid gland - T2N2. What method oftreatment is optimal in the given case?

A. Combined methodB. Surgical extraction of neoplasmC. Radiation therapyD. ChemotherapyE. Lymph nodes removal

46. A 20-year-old male patient complainsof acute pain during eating, bad breath,general weakness. Objectively: interdentalpapillae and marginal gingiva are coveredwith dirty gray film, any touch is painful. Thebody temperature is of 37, 5oC. Blood count:RBC- 4, 8 · 1012/l, Hb- 150 g/l, colour index- 0,9, WBC- 10, 5 · 109/l, JG- 2%, BASO-1%, EOS- 5%, segmented neut. - 8%, stabs- 47%, LYM- 35%, PLT- 250 · 109/l. ESR- 20mm/hour. What is the most likely diagnosis?

A. Vincent’s gingivitisB. AgranulocytosisC. Acute leukemiaD. Acute herpetic stomatitisE. Chronic leukemia

47. A 68-year-old patient, having sufferedfrom stroke, suffers from periodical bouts ofretching, which lead to unilateral dislocati-on of the temporormandibular joint. Settingthe joint was possible only with anesthesia.Objectively the following teeth are absent:from the 21 to the 28, 15, 16, 17, 18, 36, 37, 44,46, 47. What kind of denture construction wi-ll be optimal for the duration of treatment?

Page 9: Krok 2 STOMATOLOGY - Номеmmf.dsmu.edu.ua/attachments/article/24/E94t10_2016E.pdf · 2018-06-06 · Krok 2 Stomatology (англомовний варiант, iноземнi студенти)

Krok 2 Stomatology (англомовний варiант, iноземнi студенти) 2016 рiк 7

A. Removable laminar denture with mouthopening restrictorB. Schroder appliance with sliding jointC. Petrosov appliance with restrictorD. Yadrova applianceE. Khodorovych-Burgonska appliance withrestrictor

48. A 9-year-old child complains of painin the left mandibular molar that occursduring eating. The masticatory surface ofthe 75 tooth exhibits a carious cavitiy filledwith softened dentin and localized withinparapulpar dentin. The cavity communi-cates with the tooth cavity. Probing at thesite of communication causes acute painand moderate bleeding. Percussion of thetooth causes no pain response. Radiographyrevaled no periodontal alterations in the 75tooth. Select the method of treatment:

A. Devital amputationB. Devital extirpationC. Vital extirpationD. Vital amputationE. Biological method

49. A 40-year-old patient complains of feverup to 38oC, and a roundish infiltration onthe upper lip. Objectively: the upper lipon the left exhibits a round infiltrate, theoverlying skin is deep crimson. The infiltrateadheres to the surrounding tissues and has anecrotic core in the center. The upper lip ishyperemic, edematous. What diagnosis canbe made?

A. Upper lip furuncleB. Acute abscess of the upper lipC. Retention cystD. Acute lymphadenitisE. Upper lip carbuncle

50. A 25-year-old patient received a traumato the chin region. On the basis of X-ray hewas diagnosed with bilateral mental fractureof mandible. Specify the direction of di-splacement of the minor fragment:

A. Downward and backwardB. Upward and forwardC. Upward and backwardD. Downward and forwardE. There is no displacement

51. A 38-year-old patient with chronicgeneralized periodontitis has been referredto orthopedic treatment. Objectively: denti-tions are without gaps, the 12, 11, 21, 22teeth are pulpless and exhibit I grade mobi-lity. The other teeth are stable. What is themost aesthetic splint for the anterior teeth?

A. Mamlok splintB. Ring adhesion splintC. Soldered combined crownsD. Cap splintE. Brace

52. A 42-year-old patient has been hospitali-zed with Le Fort II fracture of maxilla. Selectan appliance for the treatment of this pati-ent:

A. ZbarzhB. KulaginC. RudkoD. Penn-BrownE. Yadrova

53. A 25-year-old patient complains of briefpain attacks during eating sweet, hot or coldfood. Objectively: on the contact surface ofthe 36 tooth there is a carious cavity withinparapulpar dentin. The dentin is softened.Probing of the cavity is painful. Vertical andhorizontal percussion is painless. Electricpulp test - 9 microamperes. What diagnosisis most likely?

A. Acute deep cariesB. Acute median cariesC. Chronic fibrous pulpitisD. Chronic deep cariesE. Chronic median caries

54. A 39-year-old patient suffers from mi-crostomia. What impression tray should beused?

A. DemountableB. StandardC. Standard for edentulous jawD. Custom plasticE. Collapsible

55. In a prostodontic clinic a partial lami-nar denture for the upper jaw is being madefor a 53-year-old patient. Objectively: dentalformula is 14, 13, 12, 11, 21, 22, 23, 24, 27.The teeth are firm, clinical crowns are tallwith pronounced equator. X-ray shows noperiapical changes in the periodontium ofthe abutment teeth. What clammer fixationis optimal for this patient?

A. PlanarB. SagittalC. DiagonalD. TransversalE. Point

56. An intradental splint for the 33, 32, 31,41, 42, 43 teeth is being made for a 50-year-old patient. During its installation inthe oral cavity hemorrhage occurred fromthe 41 tooth root canal. What caused thehemorrhage?

Page 10: Krok 2 STOMATOLOGY - Номеmmf.dsmu.edu.ua/attachments/article/24/E94t10_2016E.pdf · 2018-06-06 · Krok 2 Stomatology (англомовний варiант, iноземнi студенти)

Krok 2 Stomatology (англомовний варiант, iноземнi студенти) 2016 рiк 8

A. Wall perforation of the root canalB. Pulp traumaC. Trauma of the gingival marginD. Damaged root bifurcationE. Crown perforation

57. A 34-year-old male patient complains ofacute spasmodic pain in the region of hisupper jaw on the left that is aggravatingwhen affected by cold stimuli. Toothacheirradiates to the ear and temple. He hadacute toothache of the 37 tooth one yearago, but he did not consult a dentist. Painrecurred three days ago. Objectively: the 37tooth has a carious cavity communicatingwith the dental cavity. Probing of the openedcarious cavity is extremely painful. X-ray pi-cture shows widening of periodontal fissureat the root apex of the 37 tooth. What is themost likely diagnosis?

A. Exacerbation of chronic pulpitisB. Exacerbation of chronic granulatingperiodontitisC. Exacerbation of chronic fibrous peri-odontitisD. Acute diffuse pulpitisE. Acute purulent pulpitis

58. A 79-year-old female patient consulted aprosthodontist about denture replacement.The patient has a history of a stroke.Objectively: acute irregular atrophy of thealveolar processes of both jaws is present;mucous membrane of the oral cavity isdry and nonmobile. The previous denturescannot be fixed. What is the most appropri-ate prosthetic construction?

A. Dentures with elastic liningB. Dentures with extended bordersC. Dentures with metal basesD. Dentures with shortened bordersE. Implant-supported dentures

59. A 56-year-old patient complains offrequent breakage of the basis of the partiallaminar denture of the upper jaw. Objecti-vely: only the 23 tooth is retained on theupper jaw. Cracks in the deture basis appearin the area of the 23 tooth. What should bedone to reinforce the denture basis?

A. Make a new denture with telescopicfixation on the 23rd toothB. Make the basis out of Ethacryl-02 plasticC. Make the basis out of Phtorax plasticD. Make the basis out of Prothacryl-M plasticE. Make the basis out of Redont plastic

60. A patronage nurse visited a newbornbaby. Examination revealed the shortenedlower part of the face, the backward-slopingchin, missing teeth, the retroposed lowerjaw. What is the number of dental folliclesin each jaw of a newborn baby?

A. 18B. 16C. 14D. 12E. 10

61. A 47-year-old patient complains of aburning sensation and pain in the mouth.Objectively: on the mucous membrane ofcheeks along the line of teeth contact andin the corners of mouth there are multi-ple polygonal bright red erosions 1,0-1,5cm in diameter located on the hyperkerati-nized plaque and opaque whitish mucosa.Cytological analysis revealed keratinizingepithelial cells. What is the most likely di-agnosis?

A. Leukoplakia, erosive formB. Lichen planus, erosive formC. Erythema multiformeD. Secondary syphilisE. Lupus erythematosus, erosive form

62. A 27-year-old female patient consulteda dentist about pain in the 35 tooth. Thepain is caused by cold stimuli and qui-ckly abates after the stimulus is removed.Objectively: in the cervical region of the 35tooth there is a dental tissue defect withinthe enamel. The defect has white crumblingedges. Electroexcitability of the pulp was 5mkA. What is the most likely diagnosis?

A. Acute superficial cariesB. Local enamel hypoplasiaC. Acute initial cariesD. Enamel necrosisE. Enamel erosion

63. A 27-year-old female patient has her 26tooth cavity accidentially breached along themesial buccal line angle during treatment ofacute deep caries of the 26. Choose the opti-mal method of treatment:

A. Biological methodB. Non-vital extirpationC. Non-vital amputationD. Vital amputationE. Vital extirpation

64. A 55-year-old patient consulted a dentistabout a roundish tumour-like formation ofabout 1 cm in diameter located within thevermilion border of his lower lip. Objecti-vely: the tumor-like formation protrudesabout 5 mm above the vermilion border, it isdense and grayish-red in color. The surfaceof the formation is covered with thin scalesthat can hardly be removed. What is themost likely diagnosis?

Page 11: Krok 2 STOMATOLOGY - Номеmmf.dsmu.edu.ua/attachments/article/24/E94t10_2016E.pdf · 2018-06-06 · Krok 2 Stomatology (англомовний варiант, iноземнi студенти)

Krok 2 Stomatology (англомовний варiант, iноземнi студенти) 2016 рiк 9

A. Verrucous precancer of the vermilionborder of lipB. Abrasive precancerous Manganotti’scheilitisC. Precancerous limited hyperkeratosis ofthe vermilion border of lipD. Bowen’s diseaseE. Erythroplasia of Queyrat

65. A 45-year-old man complains of drynessand painfulness of the lower lip. On exami-nation: the lower lip is swollen, dry, coveredin small scales and fissures. In the Klein area(wet-dry line) there are dilated openingsof salivatory glands observed as red dotsproducing clear substance. The lower lipmucosa is lumpy. What is the most likelydiagnosis?

A. Cheilitis glandularisB. Actinic cheilitisC. Meteorological cheilitisD. Exematous cheilitisE. Cheilitis exfoliativa

66. A 50-year-old patient has a defect of thelower dental arch. It is planned to make animplant-supported bridge for its restoration.X-ray picture shows that the height of thebone mass from projection of mandibularcanal up to the top of alveolar crest is 2 cm.What type of implant should be applied?

A. ThreadedB. Endodontic-endoosseousC. Plate-formD. SubperiostealE. Conical

67. A 40-year-old man, a chemical industryworker, notes the development of sorenessof the mouth, pain response to thermal andchemical stimuli. On examination: on thevestibular surface and cutting edge of thefront teeth there are enamel defects wi-th uneven scalloped margins, chalk-like incolor. Make the diagnosis:

A. Acidic necrosis of enamelB. Superficial cariesC. Enamel hypoplasia (erosive form)D. Fluorosis (erosive form)E. Median caries

68. A 57-year-old female patient complainsof burning pain, dry mouth, sensation of aforeign body on the tongue back and lateralsurfaces, which disappears in the process ofeating. The first case of such symptoms wasnoted a year ago after psychological trauma.The patient suffers from pancreatic di-abetes and sleep disturbance. Examinationrevealed the following: the tongue mucosais without significant changes, moderatelymoistened. What is the most probable di-agnosis?

A. GlossodyniaB. Candidal glossitisC. Glossitis areata exfoliativaD. Hunter-Moeller’s glossitisE. Glossitis with fissured tongue

69. A patient presented to a dental cli-nic for complex dental care. Objectively:the 37 tooth exhibits a deep carious cavi-ty communicating with the tooth cavity.There is no response to stimuli. Radiographyreveals widening and deformation of theperiodontal ligament space in the apicalregion. What is the most likely diagnosis?

A. Chronic fibrous periodontitisB. Chronic granulating periodontitisC. Chronic granulomatous periodontitisD. Chronic fibrous pulpitisE. Chronic gangrenous pulpitis

70. An 18-year-old patient complaining oflarge diastem has made an appointmentwith a prosthodontics specialist. Objecti-vely: there is full lateral displacement of thecentral incisors due to absence of the 12 and22 teeth. What instrument is the most advi-sable for moving the central incisors closertogether?

A. Korkhaus applianceB. Vasylenko applianceC. Simple cotton ligatureD. Kalvelis applianceE. Begg appliance

71. An injured patient complains of limi-ted opening of the mouth, nose bleedi-ng, skin numbness in the infraorbital andlower eyelid region. Objectively: there isface deformation due to the depression ofsoft tissues in the left cheekbone region, stepdeformity in the middle part of the inferiormargin of the left orbit and in the area of thezygomatic alveolar crest. What is the most li-kely diagnosis?

A. Zygomatic bone fracture with di-splacement of bone fragmentsB. Fracture of the right zygomatic bonewithout displacement of fragmentsC. Le Fort I fracture of maxillaD. Le Fort II fracture of maxillaE. Fracture of the malar arch

72. A 60-year-old patient addressed a denti-st with complaints of a conic protrusionappearing on the skin of his chin on theleft. Height of the protrusion is 1,5 cm, wi-dth - 0,8 cm. Palpation is painless; there areclear margins; the protrusion is gray-brownin color; on palpation of the soft tissuesnear the protrusion base a thickening canbe detected. What preliminary diagnosis canbe made?

Page 12: Krok 2 STOMATOLOGY - Номеmmf.dsmu.edu.ua/attachments/article/24/E94t10_2016E.pdf · 2018-06-06 · Krok 2 Stomatology (англомовний варiант, iноземнi студенти)

Krok 2 Stomatology (англомовний варiант, iноземнi студенти) 2016 рiк 10

A. Cutaneous hornB. Bowen’s diseaseC. PapillomaD. KeratoacanthomaE. Melanoma

73. A 30-year-old patient needs to have his26 tooth extracted because of exacerbati-on of chronic periodontitis. Objectively: thecrown of the 26 tooth is decayed by 1/3.What forceps can be used for this toothextraction?

A. S-shaped forceps with a projecting tip onthe left beakB. S-shaped forceps with a projecting tip onthe right beakC. Straight forcepsD. Straight elevatorE. S-shaped forceps without projecting tips

74. A patient complains of restricted mouthopening, pain during swallowing, fever upto 38, 5oC, weakness, indisposition. Objecti-vely: the mouth opens up to 1 cm. AfterBerchet anesthesia examination of theoral cavity revealed edema, hyperemia,tenderness of the pterygomandibular fold.Lateral and posterior pharyngeal wallsare intact. The 38 tooth is semi-impacted.Palpation of the internal surface of themandibular angle is painful. What is themost likely diagnosis?

A. Phlegmon of the pterygomandibularspaceB. Acute tonsillitisC. Phlegmon of the infratemporal regionD. Acute pericoronitis of the 38 tooth regionE. Phlegmon of the peripharyngeal space

75. A 37-year-old patient complains ofbleeding gums, bad breath, loose teeth, di-fficulties with chewing food. Objectively:gums are bluish-red in colour; periodontalpockets are 6 mm deep and contain seroussubstance; class II teeth mobility; moderatedeposits of subgingival and supragingivaldental calculus; Oral Hygiene Index equals 3points; there is traumatical occlusuion alongall the length of the dental arches. What isthe provisional diagnosis?

A. Generalized periodontitis, stage II,chronicB. Generalized periodontitis, stage I, acuteC. Hypertrophic gingivitisD. Histiocytosis XE. Generalized periodontitis, stage II, acute

76. A 3-year-old child suffers from extremegingivitis with slight teeth mobility; the teethare intact. The skin is dry, the hair is brittle,the skin of the palms and soles of the feet isrough and covered in small cracks. What is

the provisional diagnosis?

A. Papillon-Lefevre syndromeB. Niemann-Pick diseaseC. Hand-Schuller-Christian diseaseD. Eosinophilic granuloma (Taratynov’sdisease)E. Letterer-Siwe disease

77. A 49-year-old patient has a toothremoved from the left upper jaw underplexus anesthesia with articaine-forte (Arti-caine + Epinephrine). After the operationthe tooth socket did not fill with blood clot.How to prevent alveolitis in this case?

A. Loosely fill the socket with iodoformtamponB. Fill the socket with hemostatic spongeC. Lavage the socket with microcide solutionD. Lavage the socket with 0,1% chlorhexidi-ne solutionE. Fill the socket with antibiotic dustingpowder

78. A 24-year-old woman consulted a denti-st about pain in the 26 tooth. After a physi-cal examination the patient was diagnosedwith exacerbation of chronic periodontitisof the 26 tooth. During the tooth extracti-on the coronal part of the tooth was acci-dentally broken. Further manipulations wi-th bayonet forceps failed to extract the toothroots. What actions are to be taken for thesuccessful root extraction?

A. To disjoin the roots by using a drill andfissure burrB. To send the patient to the hospitalC. To use Lecluse elevatorD. To complete the tooth extraction duringthe next visitE. To use angular elevator

79. Preventive examination of a 5-year-oldchild revealed a habit of lower lip biting.What malocclusion may develop if the childkeeps this habit?

A. Anterior biteB. Prognathic biteC. Open biteD. Deep overbiteE. Cross-bite

80. A 7-year-old child has protruding chin,the lower lip overlaps the upper one. Thereare diastemas and tremas between the lowerincisors, the lower incisors overlap the upperincisors by 2/3 of the crown height. Fi-rst permanent molars demonstrate Angle’sclass III relation. Sagittal gap is 3 mm. Thecorrect doctor’s tactics will be to:

Page 13: Krok 2 STOMATOLOGY - Номеmmf.dsmu.edu.ua/attachments/article/24/E94t10_2016E.pdf · 2018-06-06 · Krok 2 Stomatology (англомовний варiант, iноземнi студенти)

Krok 2 Stomatology (англомовний варiант, iноземнi студенти) 2016 рiк 11

A. Use Bruckl’s applianceB. Recommend a complex of myogymnasticexercisesC. Use Angle’s apparatusD. Use Bynin applianceE. Use Schwartz appliance

81. A 5-year-old child was found to havemissing upper molars. Lower incisors arein contact with the mucous membrane ofpalate. Specify the doctor’s tactics:

A. Fabricate a removable laminar dentureB. Examine the child every six months untilthe eruption of permanent teethC. Examine the child once a year until theeruption of permanent teethD. Fabricate an orthodontic appliance for thetreatment of closed biteE. Medical intervention is not needed

82. A 14-year-old girl complains of indisti-nct pronunciation that showed up at theage of 14 after the acute respiratory vi-ral disease. Examination revealed normalface and normal teeth alignment, occlusaldisharmony was not found. Palpation didnot reveal cleft palate. Uvula does not moveduring pronunciation of sounds, its palpati-on does not cause gag reflex. What is thecause of indistinct pronunciation of sounds?

A. Paresis of the soft palate and uvulamusclesB. Adenoid vegetationsC. Palatal slitD. Hypertrophy of lingual tonsilE. Deformation of the bite

83. A 42-year-old woman has madean appointment with a prosthodonticoffice to make a denture. Objecti-vely the dental formula is as follows:

18 . . . . 13 12 11 21 22 23 . . . . 2848 47 46 45 44 43 42 41 31 32 33 34 35 36 37 .

The patient has deep occlusion; clinicalcrowns are low; equator is not pronounced.The patient suffers from epileptic seizures.What kind of denture should be prescribedfor this patient?

A. Partial removable laminar metal-baseddentureB. Dental bridgeC. Partial removable laminar plastic denturewith clasp-retainersD. Partial removable laminar denture withsupporting-retaining claspsE. Bugel dental prosthesis

84. A 15-year-old girl complaining ofmoderate swelling of the left parotic-masticatory area is being examined by adental surgeon. Palpation revealed the leftparotid gland to be bulgy, dense, and pai-nless. Turbid saliva is being secreted from

the duct. The duct orifice is dilated, thesurrounding mucosa is cyanotic, pastose, wi-th teeth imprints. What disease is it?

A. Chronic parenchymatous sialoadenitisB. Chronic interstitial sialoadenitisC. SialolithiasisD. Mixed tumorE. Cyst

85. A 39-year-old patient complains ofexperiencing pain in the region of the 21tooth for 2 days. It is known from the historythat the indicated tooth had been treatedbefore for caries. Objectively: the 21 tooth iscovered with metal-ceramic crown, mucousmembrane in apex projection is edematicand hyperemic. Percussion of the toothis extremely painful. X-ray picture showsimproperly filled root canal. It is planned toremove the 21 tooth crown. What kind ofanesthesia should be administered?

A. Field block anaesthesiaB. Infiltration anaesthesiaC. Intraligamentous anaesthesiaD. Application anaesthesiaE. Intraosseous anaesthesia

86. At a prosthetics dentistry clinic a 35-year-old patient received a porcelain-fused-to-metal crown for the 21 tooth. What isthe minimum warranty period for porcelain-fused-to-metal crowns under the currentlegislation?

A. 12 monthsB. 24 monthsC. 36 monthsD. 6 monthsE. 3 months

87. During the preparation of a tooth a pati-ent had an epileptic seizure. The seizure wasarrested. What mistake did the orthopaedistmake?

A. Did not collect complete history dataB. Did not apply one of the types of localanesthesiaC. Violated the rules of preparationD. Skipped psychological preparation of thepatientE. Did not apply general anaesthesia

88. During the surgical removal of a retenti-on cyst of the lower lip a 14-year-old boycomplained of sudden weakness, dizziness,nausea. Objectively: the skin is covered withcold sweat. Respiration is frequent, pulse isweak, BP is low (90/60 mm Hg), the handsare cold. What is the most likely diagnosis?

Page 14: Krok 2 STOMATOLOGY - Номеmmf.dsmu.edu.ua/attachments/article/24/E94t10_2016E.pdf · 2018-06-06 · Krok 2 Stomatology (англомовний варiант, iноземнi студенти)

Krok 2 Stomatology (англомовний варiант, iноземнi студенти) 2016 рiк 12

A. SyncopeB. Toxic shockC. Traumatic shockD. Anaphylactic shockE. Collapse

89. A 56-year-old patient addressed a dentalsurgeon for extraction of the 27 tooth. Whatanesthesia should be applied?

A. Tuberal, palatalB. Palatal, infraorbitalC. Infiltration, incisiveD. MandibularE. Torusal

90. A 28-year-old patient has been sufferingfrom gum discomfort, gingival hemorrhagesand pain, especially during eating, for aweek. Objectively: the gums are swollen andmarkedly hyperemic, especially in the areaof the 43, 42, 41, 31, 32, 33 teeth, soft plaqueand tartar are present, gingival sulcus is upto 2 mm deep. What is the most likely di-agnosis?

A. Acute deep gingivitisB. Chronic catarrhal gingivitisC. Hypertrophic gingivitisD. Initial stage of chronic generalized peri-odontitisE. Exacerbation of initial generalized peri-odontitis

91. An 18-year-old patient complains ofchalky spot in the 23 tooth, which is sli-ghtly painful when cold stimulus is appli-ed. Objectively: the vestibular surface neardental cervix of the 23 tooth has a whitespot 2-3 mm in size. Probing is painless andreveals coarseness of the surface. Thermalstimulus causes slight pain. What is the mostprobable diagnosis?

A. Acute initial cariesB. Chronic initial cariesC. FluorosisD. Local hypoplasiaE. Enamel necrosis

92. A 12-year-old child presents with bodytemperature of 38oC, chills, nausea, vomiti-ng, delirium, weakness. In the middle thirdof the face there is butterfly-shaped rash.Regional lymph nodes are enlarged, sli-ghtly painful. Blood count: WBC- 12 · 109/l,lymphocytes - 8, 0 · 109/l, ESR- 26 mm/hour.Make the diagnosis:

A. ErysipelasB. Facial vein phlebitisC. Cutaneous actinomycosisD. Streptoderma. SubmandibularlymphadenitisE. Acute nonodontogenic sinusitis

93. Objective examination of a 65-year-old patient with the completely edentulousmandible revealed a marked uniformatrophy of the alveolar bone; bony promi-nences on the lingual surface in the regi-on where premolars had previously been.Mucosa was unevenly pliable; alveolar crestwas mobile in the frontal region. The clinicalcondition of the mandible should be takeninto consideration at the following stage ofdenture fabrication:

A. Taking a differentiated impressionB. Taking an anatomical impressionC. Taking a positive pressure impressionD. Taking a decompression impressionE. Measuring the centric relation of jaws

94. A 62-year-old patient diagnosed withTMJ arthritis presented to a clinic. The pati-ent underwent radiography. Specify the radi-ological presentations of TMJ arthritis:

A. Changed shape of the bone elements ofthe jointB. Joint space narrowingC. Joint space wideningD. No joint spaceE. Compaction of the articular head cortex

95. A 45-year-old patient has a bilateralmental mandibular fracture with a typicaldisplacement of fragments. What actionsshould be taken to eliminate asphyxia?

A. Pull out and pierce the tongueB. Artificial pulmonary ventilationC. Tracheal intubation through the noseD. Tracheal intubation through the mouthE. Tracheostomy

96. A 53-year-old patient complains ofabsent 12, 11, 21 and 22 teeth. On exami-nation it is decided to make a porcelain-fused-to-metal bridge with the 13 and 23 asabutment teeth. During preparation ledgesare created in abutment teeth. Ledge widthcan vary within the following range:

A. 1,5-2,0 mmB. 0,5-0,8 mmC. 2,0-2,5 mmD. 2,5-3,0 mmE. 0,1-0,2 mm

97. A 16-year-old girl complains of cosmeticdefect of the front teeth - there are darkspots and hard tissue defects. The spotswere detected in the process of teething, thedefects developed later. The following di-agnosis was made: erosive form of fluorosisof the 16, 11, 12, 22, 26, 31, 32, 36, 41, 42, 46teeth. Choose the method of treatment:

Page 15: Krok 2 STOMATOLOGY - Номеmmf.dsmu.edu.ua/attachments/article/24/E94t10_2016E.pdf · 2018-06-06 · Krok 2 Stomatology (англомовний варiант, iноземнi студенти)

Krok 2 Stomatology (англомовний варiант, iноземнi студенти) 2016 рiк 13

A. Restoration treatmentB. Remineralization treatmentC. ProstheticsD. Surgical treatmentE. Regular medical check-ups

98. Parents of a 6-year-old boy complain ofhis high body temperature, decrease of hisgeneral well-being, painful swelling of theright cheek. Objectively: the skin is pale, theface is asymmetric due to swelling of thesoft tissues of the right cheek. The maxillaryalveolar process is bilaterally thickened inthe area of the 55, 54 and 53 teeth, pai-nful on palpation, these teeth are partiallydestroyed by caries, mobile (II-III degree),their dentogingival pockets produce pus.What is the most likely diagnosis?

A. Acute odontogenic osteomyelitis from the53, 54, 55B. Acute odontogenic purulent maxillarperiostitis from the 53, 54, 55C. Odontogenic buccal abscess from the 53,54, 55D. Acute odontogenic albuminous maxillarperiostitis from the 53, 54, 55E. Odontogenic buccal phlegmon from the53, 54, 55

99. In the process of the 26th toothextraction a 34-year-old patient suddenlydeveloped the following symptoms: tinni-tus, weakness, paleness of skin. Vertigo wasdiagnosed. What is the most advisable firstaid approach?

A. Put the patient in the TrendelenburgpositionB. Intramuscular injection of 50% analgin(mеtamisole sodium) solutionC. Intravenous injection of 0,1% adrenalineD. Subcutaneous injection of cordiaminE. Intravenous injection of prednisolone

100. A 53-year-old patient complains of anulcer on the lateral surface of the tongue.The ulcer appeared 6 months ago in theresult of a trauma caused by sharp tip of the37 tooth metal crown. A dentist replacedthe crown with the one of better qualityand prescribed keratoplastic drugs. Despitethese measures the ulcer continues to grow.Lately there has been pain during talking,chewing, swallowing; sometimes the painirradiates to the pharynx. Objectively: onthe lateral surface of the tongue there isa painful ulcer with uneven raised densemargins and lumpy floor covered with grayi-sh necrotic coating. What diagnosis is mostlikely?

A. Cancer of the tongue lateral surfaceB. Trophic ulcerC. Traumatic ulcerD. Vincent’s necrotizing ulcerative stomatitisE. Tuberculous ulcer

101. An 8-year-old child has been clini-cally diagnosed with exacerbation of chronicperiodontitis of the 84 tooth. The crown isdecayed by 1/2. What is the dentist’s optimaltactics?

A. ExtractionB. Endodontic treatmentC. Endodontic treatment and drug therapyD. Drug therapyE. Opening along the mucogingival fold,drug thrapy

102. A laminar denture for the lower jaw isbeing made for a 54-year-old patient. Baseplate wax is used during laboratory stagefor wax templates. What group of accessorymaterials does such wax belong to?

A. ModelingB. AbrasiveC. FixingD. ImpressionE. Forming

103. A 28-year-old patient has beenscheduled for the surgical resection ofthe root apex of the 12 tooth for chronicgranulomatous periodontitis. What kind ofanesthesia should be administered beforethe surgery?

A. Bilateral infraorbital and incisiveB. Infraorbital and palatineC. Plexual and palatineD. Tuberal and palatineE. Terminal and incisive

104. A 11-year-old child complains of painduring eating food, especially hot, in thelower right lateral tooth. On the masti-catory surface of the 46 tooth there is alarge carious cavity filled with softenedlight-brown dentin. The cavity is located wi-thin parapulpar dentin. In the projection ofmedial buccal pulp horn the carious cavitycommunicates with the pulp chamber. Deepprobing is painful. Electric pulp test - 60 mi-croamperes. Make the diagnosis:

A. Chronic gangrenous pulpitisB. Chronic hypertrophic pulpitisC. Acute diffuse pulpitisD. Chronic fibrous pulpitisE. Acute focal pulpitis

105. A 34-year-old male patient complainsof a cosmetic defect, a cavity on the vesti-bular surface in the cervical part of the 21tooth. Objectively: the carious cavity is wi-thin the enamel, the floor and the walls are

Page 16: Krok 2 STOMATOLOGY - Номеmmf.dsmu.edu.ua/attachments/article/24/E94t10_2016E.pdf · 2018-06-06 · Krok 2 Stomatology (англомовний варiант, iноземнi студенти)

Krok 2 Stomatology (англомовний варiант, iноземнi студенти) 2016 рiк 14

pigmented, probing and percussion and pai-nless. There is no pain reaction to stimuli.What is the most likely diagnosis?

A. Chronic surface cariesB. Acute surface cariesC. Necrosis of dental hard tissuesD. Acute median cariesE. Chronic median caries

106. A 12-year-old boy complains of dullache in the upper right tooth. The painaggravates during biting. Objectively: inthe 16 tooth there is a deep carious cavitypenetrating into the tooth cavity, cold testand probing are painless, mucosa is swollen,hyperemic. X-ray reveals areas of bone ti-ssue destruction with blurred margins nearroot apices. What diagnosis is most likely?

A. Exacerbation of chronic granulatingperiodontitisB. Exacerbation of chronic granulomatousperiodontitisC. Exacerbation of chronic fibrous peri-odontitisD. Acute purulent pulpitisE. Acute purulent periodontitis

107. To fill a medium depth carious cavi-ty in the 37 tooth (Black class II) of a 35-year-old male patient a doctor has chosen atechnique of layer-by-layer tooth restorati-on. What composite material should beapplied for covering the carious cavity wallsand floor to create the initial super adaptivelayer?

A. FlowableB. CondensableC. MacrofilledD. MicrohybridE. Microfilled

108. A patient suffered a facial trauma.On examination and X-ray the patient wasdiagnosed with fracture of the maxillaralveolar process. What treatment would bemost advisable in this case?

A. Smooth oral splintB. Weber splintC. Gunning splintD. Intermaxillary ligatureE. Zbarzh apparatus

109. A 20-year-old male patient complainsof spontaneous pain in the 24 tooth, whi-ch arose suddenly and persists for about15 minutes at a time. Objectively: the distalsurface of the 24 tooth exhibits a deep cari-ous cavity with overhanging walls. The cavi-ty is filled with light-colored softened dentinand communicates with the tooth cavity. Thecold stimulus causes acute, slowly abatingpain. Percussion causes no pain response.

Select the best method of treatment:

A. Vital extirpationB. Vital amputationC. Biological methodD. Devital amputationE. Devital extirpation

110. A 9-year-old girl has been sufferingfrom gingival hemorrhages, cracks in themouth angles for a month. She has a hi-story of frequent nosebleeds, rapid fatigabi-lity. Objectively: the skin of the face andoral mucosa are pale. In the mouth anglesthere are cracks reaching the red border. Gi-ngiva in the region of the 11 and 12 teethis hyperemic and edematous, bleeds whentouched. DMF/dmf = 10. Back of the tongueis bright red, smooth, glossy. Submandi-bular lymph nodes are slightly enlarged,mobile, painless. What specialist should beconsulted in the first place?

A. HematologistB. NeuropathistC. EndocrinologistD. Infectious disease specialistE. Gastroenterologist

111. A 25-year-old patient complains of painwhen biting on the 15 tooth. The pain arosetwo days ago, has a constant aching natureand increased significantly over the last day.Objectively: the crown of the 15 tooth is gray,the medial contact surface exhibits a deepcarious cavity communicating with the toothcavity. Percussion causes acute pain, the gi-ngival mucosa in the projection of the 25tooth root apex is hyperemic. The regionallymph node is tender. Radiograph shows anill-defined zone of periapical bone destructi-on. What is the most likely diagnosis?

A. Exacerbation of chronic periodontitisB. Acute serous periodontitisC. Acute suppurative periodontitisD. Chronic granulating periodontitisE. Acute serous periodontitis, intoxicationstage

112. A 27-year-old patient has beenreferred by a prosthodontist for endodontictreatment of the 45 tooth. Objectively: the45 tooth crown is destroyed; the lateralsurface of the tongue and the buccal mucosahave patches of grayish macerated epi-thelium slightly rising above the mucosasurface at the points of direct contact wi-th the 45 tooth. The uvula and palati-ne arches are stagnant-red in color; hardpalate has papulae surrounded with redmargin and covered in grayish epithelium.The submandibular, cervical, supraclavi-cular, and subclavicular lymph nodes areenlarged and painless. What is the provisi-onal diagnosis?

Page 17: Krok 2 STOMATOLOGY - Номеmmf.dsmu.edu.ua/attachments/article/24/E94t10_2016E.pdf · 2018-06-06 · Krok 2 Stomatology (англомовний варiант, iноземнi студенти)

Krok 2 Stomatology (англомовний варiант, iноземнi студенти) 2016 рiк 15

A. Secondary syphilisB. Chronic recurrent aphthous stomatitisC. Lupus erythematosus, patch stageD. Soft leukoplakia (leucoplakia mollis)E. Lichen ruber planus

113. A 20-year-old male patient with insulin-dependent diabetes mellitus has been givenanesthesia with Ultracain DS-forte for surgi-cal oral sanitation. What complications maydevelop in this case?

A. Abnormal levels of blood glucoseB. Increased blood pressureC. ClonusD. Atopic dermatitisE. Toxic shock

114. As a result of an injury the 51and 61teeth of a 3-year-old child have completelycut into the alveolar bone. Medical historyrecord: the teeth were partially decayed,changed in colour. Select an appropriatetreatment tactics:

A. Extraction of the injured teethB. Follow-up as the teeth may erupt againC. Reposition of teeth, endodontic treatmentD. Extraction of teeth, endodontic treatment,repositionE. Treatment is not required

115. A 5-year-old girl complains of painin the mouth angles occurring duringmouth opening. She has a history of acutebronchitis and prolonged antibiotic therapy.Objectively: the red border is dry, the mouthangles are downturned. In the mouth anglesthere are cracks covered with white plaqueand surrounded by slightly hyperemicelevation, painful on palpation and mouthopening. Oral mucosa exhibits no pathologi-cal changes, decay-missing-filled index - 2,hygiene index - 2,0. What is the most likelydiagnosis?

A. Mycotic cheilitisB. Streptococcal cheilitisC. Meteorological cheilitisD. Glandular cheilitisE. Exfoliative cheilitis

116. A 43-year-old patient complains ofmobility and significant neck exposure ofthe lower front teeth. Objectively: the gumsin the region of the 44, 43, 42, 41, 31, 32, 33,34 teeth are pale and cyanotic, non-bleeding.The 42, 41, 31, 32 teeth exhibit the I-II grademobility. The overcrowding of the 42, 41,31, 32 teeth is present. The necks of the 42,41, 31, 32 teeth are exposed by 1/2 of theroot length, the necks of the 43, 33 teethare exposed by 1/4. What kind of dentalprosthesis should be applied in this case?

A. Cast removable splintB. Kurlyandsky splint barC. Cap splintD. Portion crownE. Half-ring splint

117. A patient complains of fever up to 38oC,headache, pain in the joints anf muscles,vesicles in the oral cavity, mainly in thefrontal part. Eating is sharply painful. Forthe last several years the disease recurredduring wet anf windy weather. The patientoften suffers from cases of URTI. Objecti-vely: on the buccal, lingual, labial mucosathere are confluent erosions against theerythematous background, covered in gray-white coating. There are bloody scabs on thered border and in the angles of the mouth.Make the diagnosis:

A. Erythema multiformeB. SyphilisC. Acute aphthous stomatitisD. Acute necrotizing ulcerative stomatitisE. Chronic recurrent aphthous stomatitis

118. A 14-year-old adolescent complains ofbleeding and pain during eating occurringin the 36 tooth. Anamnesis states intensepain in the past. Objectively: the crown isseverely damaged; carious cavity is large,penetrates into the tooth cavity, and is parti-ally filled with overgrown pulp. The tissuesdemonstrate low sensitivity to touch, bleedupon touching. There are significant softdental deposits on the 35 and 34 teeth. Makethe diagnosis:

A. Chronic hypertrophic pulpitisB. Chronic gangrenous pulpitisC. Chronic papillitisD. Chronic simple pulpitisE. Chronic granulating periodontitis

119. The 47, 46, 45, 35, 36, 37, 38 teeth of a57-year-old patient are missing. It is plannedto make a clasp denture. The 48 tooth incli-nes to the lingual side and forwards. On thelingual side of the 48 tooth the border lineis diagonal, on the buccal side it runs on alevel with gingival edge. What type of Ney’sclasp should be applied?

A. V type claspB. I type claspC. IV type clasp (reverse back-action)D. I-II type claspE. II type clasp

120. A 12-year-old boy due to trauma ofthe 44 and 45 teeth area suffers from thefollowing: pathologic displacement of themandibular alveolar process; rupture of thealveolar process mucosa. What additionalexamination is necessary to specify the di-agnosis?

Page 18: Krok 2 STOMATOLOGY - Номеmmf.dsmu.edu.ua/attachments/article/24/E94t10_2016E.pdf · 2018-06-06 · Krok 2 Stomatology (англомовний варiант, iноземнi студенти)

Krok 2 Stomatology (англомовний варiант, iноземнi студенти) 2016 рiк 16

A. X-ray of the mandibula in frontal andlateral projectionsB. X-ray of the cranium in axillary projectionC. X-ray of the mandibula in frontal andParma projectionD. Tomography of the mandibulaE. -

121. A 5-year-old child has sustained atrauma. Objectively: the crowns of the 51and 61 teeth are shorter then the adjacentcrowns. The mucosa surrounding the 51and 61 teeth is hyperemic and swollen. X-ray imaging shows the periodontal fissureto be absent in the apical area of the 51and 61 roots, the 51 and 61 roots apexesare submerged in the spongy bone of thebody of the lower jaw. Choose the optimaltreatment:

A. Extraction of the 51st and 61st teethB. Splinting of teethC. Regular medical check-upsD. Anti-inflammatory therapyE. Resection of the 51st and 61st teeth andtheir splinting

122. During teeth preparation a pati-ent developed an epileptic attack. Whatmeasures should a prostodontist take?

A. To take measures to prevent tongue bitingB. To continue workingC. To call in an emergency aid teamD. To measure blood pressureE. To introduse an anesthetic

123. A 34-year-old dental patient has anindication for the extraction of the 38tooth for chronic fibrous periodontitis. Thedental surgeon used torusal (Weisbrem’s)anesthesia. What nerves are blocked by thisanesthesia?

A. Superior alveolar, lingual, buccalB. Lingual, buccalC. Superior alveolar, buccalD. Superior alveolar, lingualE. Lingual, buccal, mental

124. A 26 year-old woman suffers frombleeding gums. For 3 years she has beensuffering from pancreatic diabetes. Objecti-vely: gingival papillas of all the teeth arehyperemic, swollen and overlap with theteeth crowns by 1/3 of the crowns hei-ght. What method of examination must beconducted for differentiated diagnostics?

A. X-ray radiographyB. Schiller-Pysarev iodine testC. Capillary fragility testD. RheoparodontographyE. Stomatoscopy

125. An orthodontist has been addressedby parents of a 5-year-old child. The chi-ld has the 54 tooth extracted, all the otherdeciduous teeth are present. The doctormade a thin-wall crown for the 55 tooth withinterdental wedge to the 53 tooth. What isthe purpose of such treatment?

A. Prevention of dentition malformationB. Aesthetical restorationC. Restoration of masticatory efficiencyD. Acceleration of permanent tooth eruptionE. Deceleration of permanent tooth eruption

126. A 30-year-old patient, a lecturer,complains of periodical dislocations of thelower jaw. Objectively: direct occlusion,continuous dentition is retained. Duringmouth opening there is a clicking soundin the temporomandibular joint. Whatapparatus can prevent recurrent dislocati-on?

A. Petrosov apparatusB. Oxman apparatusC. Limberg apparatusD. Shur apparatusE. Zbarzh apparatus

127. A 37-year-old patient complains ofsevere pain in the gums, difficult chewing,smell of rot from the mouth, bleeding gums,along with general fatigue, headaches, bodytemperature up to 37, 9oC. Objectively: largeamout of soft dental deposit, shape of thegingival margin is abnormal, the gingivalmargin is covered with gray coating, regi-onal lymph nodes are painful on palpation.What is the most likely diagnosis?

A. Ulcerative gingivitisB. Chronic recurrent aphthous stomatitisC. Chemical traumaD. Herpetic stomatitisE. Thermal trauma

128. Parents of a 5-year-old boy addresseda dental surgeon with complaints ofhemorrhage occurring from the boy’smouth. According to the mother the boyhad accidentally bit his tongue. Objectively:the tongue apex is damaged, the wound is5 mm deep and is bleeding profusely. Whataid should be provided?

A. Perform primary surgical treatment of thewound, apply blind suturesB. Prescribe antibacterial treatment andantiseptic oral rinsingC. Perform antiseptic treatment of the woundand stop the hemorrhage by pressing thetissuesD. Stop the hemorrhage by ligation of thelingual arteryE. Prescribe hemostatics intravenously

Page 19: Krok 2 STOMATOLOGY - Номеmmf.dsmu.edu.ua/attachments/article/24/E94t10_2016E.pdf · 2018-06-06 · Krok 2 Stomatology (англомовний варiант, iноземнi студенти)

Krok 2 Stomatology (англомовний варiант, iноземнi студенти) 2016 рiк 17

129. A 49-year-old patient was hospitali-sed to an oral surgery department with agunshot wound of the left side of the face.There is a 3,5 cm long defect of the bodyof the mandible on the left. After the ini-tial surgical d-bridement and stabilizationof the patient’s general state it is plannedto perform plastic correction of the defect.What treatment method is optimal in the gi-ven case?

A. Shvyrkov’s compression-distractionosteosynthesisB. Osteosynthesis with titanium reconstructi-on plateC. Rudko’s apparatus applicationD. Galmosh polyfunctional apparatus appli-cationE. Substitution of the defect with an autograft

130. Before performing a surgery a dentalsurgeon treats his hands according toSpasokukotsky-Kochergin method. Whatantiseptic is basic in this kind of treatmentand how is it applied?

A. 0,5% liquid ammonia solution in twowashbasins, 3-5 minutes in eachB. Tissue with 98% ethanol, 5 minutesC. Two tissues with 96% ethanol, 5 minuteseachD. Two tissues with 5% liquid ammoniasolution, 5 minutes eachE. Tissue with 0,5% chlorhexidine di-gluconate, 5 minutes

131. A 37-year-old patient addressed adental surgeon with complaints of deteri-orating welbeing, high fever up to 38, 1oC,headache, painful swelling on the chin.Objectively: there is a dense painful infi-ltration 3x4 cm in size on the skin of thechin. The skin over the infiltration is bluishin color. There are four necrosis areas in thefocal center around the hair follicles. Thesubmental lymph nodes are enlarged andpainful on palpation. Make the diagnosis:

A. CarbuncleB. FuruncleC. ErysipelasD. Dermal actinomycosisE. Festered atheroma

132. A 43-year-old woman complains ofperiodical facial assymetry and burstingstabbing pain in the submandibular areaon the right, which increases after eati-ng spicy and sour food. Eventually theseclinical presentations disappear. The pati-ent notes periodical exacerbations. Objecti-vely: the face is symmetrical, mouth openi-ng is unrestricted, mucosa of the oral cavi-ty floor is without pathologies. In the areaof the sublingual fold a thickening of softtissues can be palpated. Excretory duct of

the submandibular gland produces no saliva.What preliminary diagnosis can be made?

A. Sialolithiasis of the submandibular glandon the rightB. Acute serous lymphadenitis of thesubmandibular area on the rightC. Abscess of the mental region on the rightD. Exacerbation of sialolithiasis of thesubmandibular gland on the rightE. Abscess of the submandibular region onthe right

133. Parents complain of painfulness andmobility of the tooth of their 4-year-old chi-ld, which developed after the impact witha wooden object. Objectively: the face isasymmetrical due to swollen tissues of theupper lip. The 51 tooth is intact, with vesti-bular displacement and the II degree mobili-ty, gums around the 51 tooth are hyperemic.What preliminary diagnosis can be made?

A. Incomplete dislocation of the 51 toothB. Complete dislocation of the 51 toothC. Contusion of the 51 toothD. Acute albuminous periostitisE. Acute purulent periostitis

134. During preventive examination of a 19-year-old man it was revealed that there arecarious lesions in the fissures of the 37 and 47teeth, sealant is fully retained in the 16 and26 but absent in the closed intact fissures ofthe 36. The 46 was extracted due to compli-cated caries. Determine the degree of cariesin this patient:

A. 3B. 2C. 4D. 5E. 6

135. A dentist performs endodontictreatment of the 31 tooth of a 62-year-oldpatient. The root canal is narrow, extremelycalcificated. Choose the optimal tool to passthrough the root canal in this case:

A. PathfinderB. Gates-glidden drillC. K-ReamerD. K-File NitiflexE. H-File

136. A crown inlay for the patient’s 46 toothis being prepared. At the first clinical stagethe cavity preparation is completed withdesign of an incline. What material can beused for the inlay?

Page 20: Krok 2 STOMATOLOGY - Номеmmf.dsmu.edu.ua/attachments/article/24/E94t10_2016E.pdf · 2018-06-06 · Krok 2 Stomatology (англомовний варiант, iноземнi студенти)

Krok 2 Stomatology (англомовний варiант, iноземнi студенти) 2016 рiк 18

A. Metal alloyB. Sinma M plasticC. PorcelainD. CompositeE. Pressed ceramics

137. A 54-year-old patient complains offrequent crunching sound in the righttemporomandibular joint, which developedone month ago. In the morning thecrunching is more frequent and decreasestowards the evening. Objectively: the face issymmetrical, the skin above the joint is wi-thout changes, the mouth opens at 2,9 mm.What diagnosis is most likely in this case?

A. ArthrosisB. Acute arthritisC. Temporomandibular joint dislocationD. Chronic arthritisE. Temporomandibular joint pain dysfuncti-on syndrome

138. On examination of a 27-year-old patientthe tip of the dental probe caught on the fi-ssures of the 36, 37, 38 teeth. Margins of theenamel defect are dark, the surface is coarse.Teeth transillumination with photopolymerlamp revealed the defect to be limited to theenamel. What diagnosis is most likely?

A. Chronic superficial cariesB. Chronic median cariesC. Acute superficial cariesD. Chronic primary cariesE. Acute primary caries

139. A 21-year-old woman addressed adoctor with complaints of bleeding gumsand foul smell from her mouth. Objectively:the gums are congestively hyperemic, withcyanotic color. X-ray revealed resorption ofinteralveolar septa up to 1/3 of their height.What is the most likely diagnosis?

A. Chronic generalized periodontitis, IdegreeB. Chronic generalized periodontitis, IIdegreeC. Acute generalized periodontitis, I degreeD. Chronic generalized periodontitis, IIIdegreeE. Acute generalized periodontitis, II degree

140. A 68-year-old patient addressed asurgical department of a dental clinic forextraction of the 45 tooth. During procedurethe patient developed burning retrosternalpain attack irradiating to the left shoulder,scapula, hand. The skin is pale, BP is 140/100mm Hg, heart rate is rapid. Skin hyperplasiacan be observed in the Zakharin-Head’szones. What emergency condition did thepatient develop?

A. Angina pectoris attackB. Heart failureC. Bronchial asthma attackD. Hypertensic crisisE. -

141. During administration of Ultracainsolution for infiltration anesthesia thecondition of a 22-year-old patient sharplydeteriorated. The patient became inert, si-lent, developed cough attacks, expiratorydyspnea, wheezing. Viscous slimy sputum isexpectorated. What urgent condition did thepatient develop?

A. Bronchial asthma attackB. Anaphylactic shockC. Hypertensic crisisD. AsphyxiaE. Collapse

142. A 2,5-year-old boy has been deliveredto an oral surgery. The patient has the upperlip cleft into three fragments, the palate iswhole. Specify the period when cheiloplastyshould be performed:

A. 6-10 monthsB. 3-5 monthsC. 12-14 monthsD. 15-19 monthsE. 20-24 months

143. A 7-year-old boy has hyperemic gums;bleeding can be detected. The teeth aremobile, with roots exposed and covered wi-th a layer of coating. Periodontal pocketsdemonstrate granulation. The child is regi-stered with a pediatrician for diabetes insi-pidus. The child’s mother states that herson’s condition has deteriorated lately. Thechild suffers from periodontal syndromecharacteristic of:

A. Hand-Schuller-Christian diseaseB. Diabetes mellitusC. Gaucher’s diseaseD. Cushing’s diseaseE. Letterer-Siwe disease

144. A 40-year-old patient addressed aprosthodontist with complaints of absentcrown of the 24 tooth due to trauma. If thecrown is broken off at the gum level, thetooth should be restored with:

A. Pivot crownB. Semi-crownC. Equator crownD. Partial dentureE. Intradental inlay

145. During extraction of the 36 tooth thetooth crown was fractured. What tool shouldbe applied to extract tooth roots?

Page 21: Krok 2 STOMATOLOGY - Номеmmf.dsmu.edu.ua/attachments/article/24/E94t10_2016E.pdf · 2018-06-06 · Krok 2 Stomatology (англомовний варiант, iноземнi студенти)

Krok 2 Stomatology (англомовний варiант, iноземнi студенти) 2016 рiк 19

A. Bayonet forcepsB. Beak-shaped forceps left sideC. Beak-shaped straight forcepsD. S-shaped forceps left sideE. S-shaped forceps right side

146. A patient after the surgery forphlegmon of the pterygomandibularspace was prescribed compresses with amembrane perforating drug. Which of thegiven below is a membrane perforatingdrug?

A. Dimexid (Dimethyl sulfoxide)B. EctericidumC. Streptocid (Sulfidine)D. LincomycinE. Oflocain

147. A 64-year-old patient complains ofspots on the mucosa of the cheeks, mouthangles, palate. Objectively: on the oral cavitymucosa there are white growths that cannotbe removed with spatula. The mucosaaround the spots is pale pink in color. Thepatient has been smoking for 40 years. Whatis the most likely diagnosis?

A. LeukoplakiaB. Lupus erythematosusC. Bowen’s diseaseD. CandidiasisE. Lichen ruber planus

148. An 8-year-old girl complains of burni-ng pain in her tongue during eating spicyand sour food. Objectively: there are redirregular-shaped spots on the tongue apexand dorsum. Filiform papillae are absent inthe affected areas. The girl states that thespots periodically grow and migrate. Makethe diagnosis:

A. Benign migratory glossitisB. Median rhomboid glossitisC. Fissured tongueD. Hairy tongueE. Raspberry tongue

149. A 70-year-old patient addressed ahospital with complaints of poorly stabi-lized complete dentures of the upper andlower jaws. What method of artificial teetharrangement is preferable in making of anew denture?

A. According to individual occlusion curvesB. According to disocclusion planesC. According to standard occlusion curvesD. According to spherical occlusion curvesE. According to prothetic occlusion planes

150. A 2,5-year-old child has fever up to38, 5oC, low appetite, rash inside the oralcavity. The disease onset was 3 days ago.Objectively: the skin of the perioral area

is covered in scarce vesicles with clearcontent. Within the oral cavity on the buccaland lingual mucosa there are sharply pai-nful erosions, 2-3 mm in size, with whitecoating and hyperemic crown. The gumsare swollen, hyperemic. The submandi-bular lymph nodes are enlarged, painful onpalpation. Make the diagnosis:

A. Acute herpetic stomatitisB. Stevens-Johnson syndromeC. Erythema multiformeD. Stomatitis with the background of infecti-ous mononucleosisE. Stomatitis with the background of chi-ckenpox

151. A 9-year-old girl complains of peeli-ng lips and sensation of dryness, strain andburning in them. Objectively: on the redborder from angle to angle of the mouthand from Klein area (wet-dry line) to thered border middle there are gray scales.The scales can be removed without erosi-ons. Make the preliminary diagnosis:

A. Exfoliative cheilitis, dry formB. Exfoliative cheilitis, exudative formC. Meteorological cheilitisD. Atopic cheilitisE. Allergic contact cheilitis

152. A patient addressed a dentalsurgeon with complaints of increased bodytemperature up to 37, 6oC, swelling of thesoft tissues, pain in the 65 tooth, whi-ch aggravates upon touching it with itsantagonist tooth. On examination there arehyperemy and smoothing out of the mucogi-ngival fold of the alveolar process in the areaof the destroyed 65 tooth. What diagnosis ismost likely?

A. Acute odontogenic purulent periostitisB. Acute serous periodontitisC. Exacerbation of chronic periodontitisD. Acute odontogenic osteomyelitisE. Parodontal abscess

153. A 17-year-old patient has been sufferingfor 2 years from periodical painful swelli-ng and fistula appearing on the skin ofgonial angle area. After cleaning the fi-stula of its purulent content pain and swelli-ng disappear leaving a small concave scar.Objectively: there is a small scar on the skinin the gonial angle area. Palpation of theoral cavity floor reveals a dense band in thebuccal soft tissues, which connects fistula wi-th projection of the 47 tooth roots. Crown ofthe 47 is destroyed. What diagnosis is mostlikely?

Page 22: Krok 2 STOMATOLOGY - Номеmmf.dsmu.edu.ua/attachments/article/24/E94t10_2016E.pdf · 2018-06-06 · Krok 2 Stomatology (англомовний варiант, iноземнi студенти)

Krok 2 Stomatology (англомовний варiант, iноземнi студенти) 2016 рiк 20

A. Migrating subcutaneous granulomaB. Festered buccal atheromaC. Mandibular tuberculosisD. Actinomycosis of the parotid-masticatoryareaE. Chronic osteomyelitis of the left mandi-bular angle

154. A 30-year-old woman addresseda dental surgeon with complaints ofuncomfortable sensation of pressure in herupper right tooth, which aggravates due tohot stimulus, and foul smell from the mouth.Objectively: there is a deep carious cavityin the 17 tooth, which penetrates into thetooth cavity. Deep probing causes severepain, percussion of the 17 tooth is painful.X-ray: there is slight widening of the peri-odontal fissure near the root apex. Electricpulp test - 70 microamperes. What final di-agnosis can be made?

A. Chronic gangrenous pulpitisB. Chronic fibrous pulpitisC. Acute purulent pulpitisD. Chronic fibrous periodontitisE. Exacerbation of chronic fibrous peri-odontitis

155. A 20-year-old patient has made anappointment with a dentist. On externalexamination the patient is pale, complai-ns of long-term fever, insomnia, weightloss of 13 kg within 5 months. The pati-ent is homeless. The therapist’s consultati-on states: all groups of the lymph nodesare enlarged, hepatolienal syndrome. Bloodtest: leukocytes - 2, 2 · 109/l. Make the di-agnosis:

A. HIV infectionB. ChroniosepsisC. TuberculosisD. Infectious mononucleosisE. Lymphogranulomatosis

156. A 45-year-old patient complains ofitching gums, pain in the area of the 43,42, 41, 31, 32, 33 teeth caused by chemi-cal and thermal stimuli. Objectively: thegums are pale, no bleeding, the roots ofaffected teeth are exposed by 0,4 mm. The43 and 33 demonstrate cuneiform defect.The teeth are intact. X-ray: regular resorpti-on of interalveolar septa at 1/4 of their hei-ght. Make the diagnosis:

A. Parodontosis, I degreeB. Generalized periodontitis, I degreeC. Catarrhal gingivitisD. Hypertrophic gingivitisE. Ulcerative gingivitis

157. An 18-year-old woman complains ofgum growth, pain and bleeding duringeating solid food. Objectively: hyperemy

and swelling of the gums, gingival marginhypertrophy of the 12, 13, 14 teeth up to 1/2of the crown height. Formalin test is pai-nless. What diagnosis is most likely?

A. Hypertrophic gingivitisB. Chronic generalized periodontitis, IIdegreeC. Catarrhal gingivitisD. Ulcerative gingivitisE. Acute generalized periodontitis, I degree

158. A 48-year-old patient has made anappointment for extraction of the 35 toothdue to exacerbation of chronic periodontitis.Specify the most advisable method of peri-pheral conduction anesthesia in this case:

A. TorusalB. InfiltrationC. Extraoral mandibularD. MentalE. Plexus

159. During application of an impressiontray to the upper jaw of a 62-year-old patientthe tray falls out when the patient opens hismouth wide. What edge of the tray shouldbe shortened?

A. In posterolateral area of the maxillarytuberosityB. Along the A lineC. In the front areaD. In the palatine torus areaE. In the buccal-alveolar fold area

160. During objective examination of thepatient’s oral cavity the lower jaw is revealedto be toothless, in the front area there is amobile band of mucosa, irregular atrophy ofthe alveolar process. What method of obtai-ning functional impression would be mostadvisable in the given case?

A. Differentiated impressionB. Decompression impressionC. Compression impressionD. Selective pressure impression duringmasticationE. Gradual pressure impression

161. A 46-year-old man complains ofconstant losing of a filling in his lower ri-ght tooth. Objectively: in the 16 tooth on theapproximal masticatory surface there is adefect of crown hard tissues at 1/3. The toothhas no discoloration, percussion is painless.What construction should be prescribed?

A. InlayB. Porcelain-fused-to-metal crownC. Semi-crownD. 3/4 crownE. Plastic crown

162. A 37-year-old patient complains of an

Page 23: Krok 2 STOMATOLOGY - Номеmmf.dsmu.edu.ua/attachments/article/24/E94t10_2016E.pdf · 2018-06-06 · Krok 2 Stomatology (англомовний варiант, iноземнi студенти)

Krok 2 Stomatology (англомовний варiант, iноземнi студенти) 2016 рiк 21

aesthetic defect. Objectively: the 13 tooth isdestroyed by 2/3. The tooth is pulpless, theroot canal is filled. How deep should be theroot canal opened for pivot crown in thispatient?

A. 2/3 of the root canalB. 1/3 of the root canalC. 3/4 of the root canalD. 1/2 of the root canalE. Full length of the root canal

163. A 7-year-old girl hit her forehead oneday ago. Several hours after the sustainedtrauma a swelling developed in the leftsuperciliary area. General condition of thechild is not disrupted. Objectively: swellingof forehead tissues spreading towards theleft eyelids; the swelling is soft, fluctuati-on sign is present. Make the preliminary di-agnosis:

A. Hematoma of the left superciliary areaB. Hemangioma of the right superciliary areaC. Fracture of the temporal boneD. Fracture of the frontal boneE. Hematic abscess of the left superciliaryarea

164. Mother of a 10-year-old girl complai-ns of a cosmetic defect of the child’s 22tooth that erupted with damaged enamel.Anamnesis states premature extraction ofthe 62 tooth due to caries complication.There is a white-yellow spot with clearmargins on the vestibular surface of the 22tooth. Enamel retains glossiness, no surfaceroughness can be detected on probing. Makethe diagnosis:

A. Local enamel hypoplasiaB. FluorosisC. Acute superficial cariesD. Chronic superficial cariesE. Systemic enamel hypoplasia

165. A 2-year-old child has suffered a teethtrauma. Objectively: the crowns of the 51and 61 teeth are by 1/3 shorter than theothers. Mucosa in the area of the 51 and 61is hyperemic, swollen. X-ray reveals absenceof the periodontal fissure in the root apexareas of the 51 and 61. What treatment tacti-cs would be optimal?

A. Regular medical check-upsB. Extraction of the 51 and 61C. Reduction of the 51 and 61D. Ligature splintE. Dental reimplantation

166. A mother of a newborn complains thatthe child cannot be breastfed. After objecti-ve examination the child was diagnosed wi-th complete submucous cleft palate. Whatsurgical procedure should be prescribed and

when is the optimal time for such a surgery?

A. Uranostaphyloplasty, 4-6 yearsB. Cheiloplasty, 4-6 monthsC. Uranostaphyloplasty, 6-7 monthsD. Staphyloplasty, 10 yearsE. Uranoplasty, 1 year

167. A 19-year-old girl addressed anoncologist with complaints of slowly growi-ng tumor-like mass on the tip of her tongue.The mass was first noticed 5-6 years ago. Thepatient requested no medical help. Objecti-vely: there is a pale pink round growth withwide pedicle on the tongue apex; the growthis painless, elastic; there are no changes ofmucosa surrounding the pedicle. Submandi-bular lymph nodes cannot be palpated.What kind of tumor is it?

A. PapillomaB. AtheromaC. FibromaD. LipomaE. Keratoma

168. A 48-year-old patient has addressed ahospital with complaints of defects in theparagingival area and slight sensitivity tothermal stimuli. Objectively: there are hardtissue defects that resemble a wedge withsmooth polished walls on precervical vesti-bular surface of the 23 and 24 teeth. Thermaltest is slightly positive. What is the most li-kely diagnosis?

A. Cuneiform defectB. Enamel necrosisC. Acute deep cariesD. Enamel erosionE. Endemic fluorosis

169. Parents of a 7-year-old child addresseda hospital with complaints of the child havi-ng no permanent teeth in the front area ofthe mandible. Anamnesis states that the fi-rst deciduous teeth erupted at the age of11 months. Objective clinical examinationrevealed the following: appearance is wi-thout changes; milk occlusion; there arephysiological diastemas and tremas; edge-to-edge incisor contact. What preliminarydiagnosis can be made according to Kalvelisclassification?

A. Retarded eruptionB. Supernumerary toothC. AdentiaD. DystopiaE. Hypoplasia

170. Parents of a 4-year-old child have madean appointment with an orthodontist forpreventive examination of the child. In theoral cavity: scissor bite is observed in thefront area in the place of inscisor contact;

Page 24: Krok 2 STOMATOLOGY - Номеmmf.dsmu.edu.ua/attachments/article/24/E94t10_2016E.pdf · 2018-06-06 · Krok 2 Stomatology (англомовний варiант, iноземнi студенти)

Krok 2 Stomatology (англомовний варiант, iноземнi студенти) 2016 рiк 22

distal surfaces of the second temporarymolars are situated in one plane. Teeth inthe dental arch are placed tightly, withoutintervals. Determine, what period of occlusi-on development is described:

A. Stability of temporary occlusionB. Formation of temporary occlusionC. Involution of temporary occlusionD. Early stage of developing occlusionE. Late stage of developing occlusion

171. A 22-year-old patient addressed anorthodontist with complaints of incorrectteeth placement on the upper jaw. Theappearance is without changes. Occlusion ispermanent. The first molar relation is of the1st type according to Angle’s classification.Analysis of the control diagnostic modelsrevealed the disturbance of the Andrews’second key of optimal occlusion of the 12and 22 teeth. The patient suffers from:

A. Angulation of the maxillary teethB. Cusp-to-fissure contact between the firstpermanent maxillary and mandibular molarsC. Maxillary teeth contactD. Axial rotation of the teethE. Spee curve concavity

172. A 15-year-old girl complains of briefpain attacks in her teeth due to chemical sti-muli. Objectively: on the contact surfaces ofthe 11, 21, 22 teeth there are enamel areasmatt white in color, with lost shine, coveredin large amount of dental deposit. Enamelis softened and can be easily chipped off wi-th excavator. Probing of lesions is painless.Percussion is painless. No reaction to coldstimuli. Make the diagnosis:

A. Acute superficial cariesB. Acute median cariesC. Acute initial cariesD. Chronic initial cariesE. Chronic superficial caries

173. A 5,5-year-old child is undergoi-ng preventive examination. No complai-nts from the patient. Objectively: in theprecervical area of buccal surface of the75 tooth there is an enamel patch with lossof natural glossiness. The enamel surface iscoarse, painless on probing. Tooth percussi-on is painless. Thermodiagnosis is negati-ve. Damaged enamel stains with 2% watersolution of methylene blue. Make the di-agnosis:

A. Acute initial cariesB. Enamel hypoplasiaC. FluorosisD. Acute superficial cariesE. Chronic initial caries

174. A 13-year-old girl undergoes preventi-

ve examination. No complaints from thepatient. Objectively: on the 36 masticatorysurface there is a carious cavity withinenamel with wide inlet. Enamel of the cavi-ty floor and walls is dense and pigmented.Probing of the carious lesion is painless. Noreaction to thermal stimuli. Tooth percussi-on is painless. The girl belongs to the I groupaccording to her health status. Make the di-agnosis:

A. Chronic superficial cariesB. Acute median cariesC. Acute initial cariesD. Chronic initial cariesE. Acute superficial caries

175. A 57-year-old patient adressed aprosthodontic clinic to have a denture madefor him. After objective examination thepatient was prescribed single-unit crownsfor the 46, 47, 36, 37 teeth. What maximalangle of a tooth stump can be allowed duri-ng preparation?

A. 3-5o

B. 5-6o

C. 10-12o

D. Stump walls should be parallel to eachotherE. 7-8o

176. A 25-year-old woman, a teacher,addressed a prosthodontics clinic withcomplaints of cosmetic defect, inability totake a bite, disrupted enunciation. The teethare lost due to trauma. Dental formula:87654300!00345678. 3!3 are firm, intact,percussion is painless. What anesthesiashould be administered for this group ofteeth in preparation for making a porcelain-fused-to-metal bridge?

A. InfiltrationB. ApplicationC. Central conduction anesthesiaD. TuberalE. Intraosseous (into spongy tissue)

177. A 58-year-old patient after examinationin a prostodontic clinic has been advised tomake a full-arch prosthesis. The treatmentplan was developed, the impressions of bothjaws were obtained, central occlusion wasdetermined with block method. What stageis next?

A. ParallelometryB. Model duplicationC. Restoration of the abutment crownsD. Frame design of a full-arch prosthesisE. Preparation for model duplication

178. A 25-year-old patient addressed ahospital and upon examination was di-agnosed with unilateral fracture of the body

Page 25: Krok 2 STOMATOLOGY - Номеmmf.dsmu.edu.ua/attachments/article/24/E94t10_2016E.pdf · 2018-06-06 · Krok 2 Stomatology (англомовний варiант, iноземнi студенти)

Krok 2 Stomatology (англомовний варiант, iноземнi студенти) 2016 рiк 23

of the mandible in the area of 43, 44 teethwith persistent vertical displacement of thebone ends. Choose the functional type ofprosthodontic apparatus required for thepatient’s treatment:

A. ReponatingB. DirectingC. FormingD. SubstitutingE. Fixing

179. A 32-year-old patient addressed a denti-st with complaints of inability to close hismouth. Objectively the mouth is half-open,the chin is protruding forwards and is di-splaced to the left. Such condition occurredafter the mouth was opened wide. What di-agnosis is most likely?

A. Anterior right-sided mandibular dislocati-onB. Anterior left-sided mandibular dislocationC. Anterior bilateral mandibular dislocationD. Posterior right-sided mandibular dislocati-onE. Posterior left-sided mandibular dislocation

180. A 37-year-old patient has symmetricalface; the mucosa in the area of the 12 toothroot apex projection is pale pink; palpationis painless; the tooth crown is destroyed by1/3; percussion is painless. X-ray: the rootcanal of the 12 tooth is filled to the apex;granuloma 4 mm in diameter surrounds theroot apex. Choose the method of surgicaltreatment:

A. Granuloma removal with root apexresectionB. Root hemisectionC. Coronary radicular tooth separationD. Root amputationE. Tooth extraction

181. During preventive examination apatient was diagnosed with precanceroushyperkeratosis of the lower lip red border.What treatment should be prescribed?

A. Surgical removal of the focus withinhealthy tissuesB. Surgical removal of the focus wi-thin healthy tissues + close-focusroentgenotherapyC. No treatment is requiredD. Surgical removal of the focus withinhealthy tissues + chemotherapyE. Palliative treatment

182. In the district dispensary center forchildren with cleft lip and palate a childwas diagnosed with congenital incompletecleft upper lip on the left with malformedcutaneo-cartilaginous portion of the nose.What dispensary group does the child

belong to?

A. FirstB. SecondC. ThirdD. FourthE. Fifth

183. Preventive examination of the oral cavi-ty of a 9,5-year-old child revealed bilateralcross bite without displacement of the lowerjaw. What biometric method of dentitionmodel investigation should be applied in thiscase?

A. Pont indexB. Gerlach indexC. Schwarz indexD. Korkhaus indexE. Tonn index

184. For a 30-year-old patient a compositeinlay for the 37 tooth is being made. Objecti-vely: there is a carious cavity of medium size(Black’s classification I class) on the masti-catory surface of the 37 tooth. What specificsare required for the tooth preparation in thiscase?

A. Creating a flat floorB. Creating an auxiliary ledgeC. Creating an inclineD. Widening of the cavity floorE. Creating an auxiliary cavity

185. A bugle denture for the lower jaw isplanned to be made for a 53-year-old pati-ent. Objectively: the 38, 37, 35, 34, 45, 46, 47teeth are absent. The retained teeth are fi-rm, with low clinical crowns. What fixationmethod of bugle denture would be optimalin this case?

A. Telescopic systemB. Supporting retentive clammersC. Beam systemD. AttachmentsE. Ball joint attachment

186. A 30-year-old patient complains ofspontaneous pain attacks in the lower lefttooth, which occurred 2 hours ago duri-ng travel by plain. Objectively: there ispathological abrasion of the 36 tooth, toothpercussion is slightly painful. After addi-tional investigation the diagnosis is made:chronic pulpitis. What investigation allowsto make such a diagnosis?

A. X-ray radiographyB. ProbingC. Thermal testsD. Luminescence diagnosticsE. Electric pulp test

187. A 22-year-old student arrived for an

Page 26: Krok 2 STOMATOLOGY - Номеmmf.dsmu.edu.ua/attachments/article/24/E94t10_2016E.pdf · 2018-06-06 · Krok 2 Stomatology (англомовний варiант, iноземнi студенти)

Krok 2 Stomatology (англомовний варiант, iноземнi студенти) 2016 рiк 24

appointment with a dentist in the afternoonafter his classes were over. The patient isregistered for regular check-ups with anendocrinologist. During treatment the pati-ent developed excited state followed by lossof consciousness. Objectively: tremor of theextremities, moist skin, heart rate - 100/min.,BP- 100/60 mm Hg. What diagnosis is mostlikely?

A. Hypoglycemic comaB. SyncopeC. Anaphylactic shockD. Hyperglycemic comaE. Epileptic attack

188. A 32-year-old patient consulted adentist with complaints of tooth crowndiscoloration on the upper right jaw.Anamnesis: 3 weeks ago the patient suffereda trauma of the upper jaw. On examination:crown of the 11 tooth is intact, gray, pai-nless on percussion, no changes of mucosain the area of root apex projection of the 11tooth. Electric pulp test - 100 microamperes.The patient was diagnosed with traumaticperiodontitis of the 11 tooth. What methodof removing tooth discoloration would beadvisable after endodontic treatment?

A. Intracrown bleachingB. External bleachingC. Custom teeth whitening with a bleachingtray (kappa)D. Microabrasion methodE. Air-abrasive method

189. A 39-year-old patient was diagnosed wi-th chronic generalized periodontitis, initialstage. After undergoing complex treatmentthe patient received instructions and wasregistered for regular check-ups. How oftenshould the patient undergo these regularcheck-ups?

A. Once every 6 monthsB. Once every 3 monthsC. Once every 9 monthsD. Once every 12 monthsE. Once every 18 months

190. A 22-year-old patient is diagnosed wi-th chronic granulomatous periodontitis ofthe 46 tooth. During the first appointmentthe patient was prescribed a temporary filli-ng made of antibacterial mateterial that sti-mulates osteo-, dentino- and cement genesis.Specify the material that satisfies this requi-rements:

A. Calcium hydroxideB. IodoformC. DexamethasoneD. ThymolE. Camphor

191. A 24-year-old patient addressed afirst-aid center with complaints of facialpain on the left, restricted mouth openi-ng. Objectively: there is moderate swellingand hematoma in the left malar arch area.On palpation there is bone tissue retracti-on and moderate painfulness. Occlusion isintact. Mouth opening is 1-1,5 cm. Uponattempt to open the mouth further there area sensation of mechanical obstructon andpain aggravation. What is the most likely di-agnosis?

A. Fracture of the left malar archB. Fracture of the left zygomatic boneC. Le Fort maxillary fracture, III gradeD. Le Fort maxillary fracture, II gradeE. -

192. A 22-year-old patient has suffered uni-lateral linear fracture in the area of thegonial angle. Immobilization was provi-ded with full dental brace with loops andintermaxillary elastic expansion. Recoverywas uncomplicated. The brace should beremoved after:

A. 3 weeksB. 2 weeksC. 1 weekD. 10 daysE. -

193. A 19-year-old woman complains ofaesthetic defects, dry lips, unpleasantsensation of strain in the lower lip and itsconstant peeling. The patient often bites herlips, rips and bites off peeling flakes, whichsometimes results in bleeding; she notes herincreased irritability. On examination thereare numerous transparent scales betweenthe red border and mucosa. The scales aretightly adjacent in the center and loose atthe periphery. They can be easily removed,produce no erosion. Regional lymph nodescannot be palpated. What diagnosis is mostlikely?

A. Cheilitis exfoliativaB. Mycotic cheilitisC. Meteorological cheilitisD. Allergic contact cheilitisE. Actinic cheilitis

194. A 73-year-old man is registered forregular check-ups in an oncological clinicafter completion of the combined treatmentfor oral mucosa cancer stage II (radiati-on therapy and surgery). During one ofthe routine check-ups an area of exposedmandibular bone is detected. There areno inflammatory changes of surroundingmucosa. A sinus tract with weak granulati-on is observed. Mandibular X-ray showssequestrum without clear margin betweenhealthy and necrotic bone. What diagnosis

Page 27: Krok 2 STOMATOLOGY - Номеmmf.dsmu.edu.ua/attachments/article/24/E94t10_2016E.pdf · 2018-06-06 · Krok 2 Stomatology (англомовний варiант, iноземнi студенти)

Krok 2 Stomatology (англомовний варiант, iноземнi студенти) 2016 рiк 25

is most likely?

A. Mandibular osteoradionecrosisB. Acute purulent mandibular osteomyelitisC. Posttraumatic mandibular osteomyelitisD. Relapse of oral mucosa cancerE. Chronic mandibular periostitis

195. A patient with odontogenic phlegmonof the oral cavity floor and neck has beendelivered into an oral surgery department.The patient’s general condition is severe,labored breating, inflammatory masticatorymuscles contracture II degree, swallowi-ng is impossible. There can be determi-ned dense sharply painful infiltration of thelower face, oral cavity floor and neck; theskin is strained and cannot be pinched in afold; fluctuation can be palpated in the oralcavity. After 30 minutes of the patient’s stayin the in-patient department the patient’sneck swelling significantly enlarged, breati-ng deteriorated. What kind of asphyxia candevelop in the patient?

A. StenoticB. DislocationalC. AspirationD. ValvularE. Obturative

196. A 16-year-old girl suffers from enlargedgums. Objectively: gingival margin of thevestibular surface of the gums is diffuselythickened, dense, pale pink in color. Gingi-val papilla cover crowns of all the teeth up to1/2 of their height, hemorrhage is absent. X-ray reveals no pathological changes of peri-odontal tissues. Make the diagnosis:

A. Chronic hypertrophic gingivitisB. Exacerbation of catarrhal gingivitisC. Acute catarrhal gingivitisD. Chronic catarrhal gingivitisE. Gingival fibromatosis

197. A 40-year-old patient complains of painin the tragus area, clicking during mouthopening, stuffed ears. Objectively: the faceis symmetrical, mouth opening path is strai-ght. Dentition defect can be estimated as theI class by Kennedy; the 18, 17, 16, 26, 27, 28teeth are absent. In this case the load wouldbe the most traumatizing for the followinganatomical structure:

A. Interarticular diskB. Articular capsuleC. Articular headD. Distal slope of the articular tubercleE. Socket floor of the temporal bone

198. A 34-year-old patient complains of painin the lower right jaw, reduced mouth openi-ng. Objectively: dentition is intact both onthe upper and lower jaw, there is prematurecontact between the 46 and 47 teeth. Di-agnosis: traumatic fracture of the mandiblein the area of the 46 tooth with fragmentdisplacement. What construction would beadvisable for temporary immobilization ofthe mandible fragments?

A. Tigerstedt’s splintB. Temporary plastic splintC. Entin’s head-chin strapD. Plastic kappaE. Weber’s splint

199. A 55-year-old patient addressed aprostodontic clinic to have a denture made.During preparation of the 44 and 47 teethfor crown installation the patient becameunwell: he developed vertigo, nausea, ti-nnitus, blurred vision, dyspnea. Unconsci-ousness followed, the pupils were dilated,heart rate was weak. After 1 minute thepatient’s condition improved. What condi-tion occurred in the patient?

A. DizzinessB. Anaphylactic shockC. CollapseD. Hypoglycemic comaE. Bronchial asthma

200. A 23-year-old woman complains ofcarious cavity present in the 27 tooth,where food particles are retained. Objecti-vely: there is a deep carious cavity on themasticatory surface of the 27 tooth filledwith dense dark-brown dentin. Probingof the cavity floor and walls is painless,thermometry and percussion are painless.Electric pulp test - 10 microamperes. X-rayreveals no pathological changes. Make thediagnosis:

A. Chronic deep cariesB. Chronic median cariesC. Acute deep cariesD. Chronic fibrous pulpitisE. Chronic periodontitis

Page 28: Krok 2 STOMATOLOGY - Номеmmf.dsmu.edu.ua/attachments/article/24/E94t10_2016E.pdf · 2018-06-06 · Krok 2 Stomatology (англомовний варiант, iноземнi студенти)

INSTRUCTIONAL BOOK

Ministry of public health of Ukraine (MPH of Ukraine)

Department of human recources policy, education and science

Testing Board

TEST ITEMS FOR LICENSING EXAMINATION: KROK 2. STOMATOLOGY.

Kyiv. Testing Board.

(English language).

Approved to print 01.03./№24. Paper size 60х84 1/8

Offset paper. Typeface. Times New Roman Cyr. Offset print.

Conditional print pages 24. Accounting publishing pages 28.

Issue. 387 copies.

Page 29: Krok 2 STOMATOLOGY - Номеmmf.dsmu.edu.ua/attachments/article/24/E94t10_2016E.pdf · 2018-06-06 · Krok 2 Stomatology (англомовний варiант, iноземнi студенти)

List of abbreviations

A/G Albumin/globulin ratio HR Heart rate

A-ANON Alcoholics anonymous IDDM Insulin dependent diabetes mellitus

ACT Abdominal computed tomography IFA Immunofluorescence assay

ADP Adenosine diphosphate IHD Ischemic heart disease

ALT Alanin aminotranspherase IU International unit

AMP Adenosine monophosphate LDH Lactate dehydrogenase

AP Action potential MSEC Medical and sanitary expert committee

ARF Acute renal failure NAD Nicotine amide adenine dinucleotide

AST Aspartat aminotranspherase NADPH Nicotine amide adenine dinucleotide

phosphate restored

ATP Adenosine triphosphate NIDDM Non-Insulin dependent diabetes

mellitus

BP Blood pressure PAC Polyunsaturated aromatic

carbohydrates

bpm Beats per minute PAS Periodic acid & shiff reaction

C.I. Color Index pCO2 CO2 partial pressure

CBC Complete blood count pO2 CO2 partial pressure

CHF Chronic heart failure pm Per minute

CT Computer tomography Ps Pulse rate

DIC Disseminated intravascular coagualtion r Roentgen

DCC Doctoral controlling committee RBC Red blood count

DM-2 Non-Insulin dependent diabetes mellitus RDHA Reverse direct hemagglutination assay

DTP Anti diphtheria-tetanus vaccine Rh Rhesus

ECG Electrocardiogram (R)CFT Reiter's complement fixation test

ESR Erythrocyte sedimentation rate RIHA Reverse indirect hemagglutination

assay

FC Function class RNA Ribonucleic acid

FAD Flavin adenine dinucleotide RR Respiratory rate

FADH2 Flavin adenine dinucleotide restored S1 Heart sound 1

FEGDS Fibro-esphago-gastro-duodenoscopy S2 Heart sound 2

FMNH2 Flavin mononucleotide restored TU Tuberculin unit

GIT Gastrointestinal tract U Unit

Gy Gray USI Ultrasound investigation

GMP Guanosine monophosphate V/f Vision field

Hb Hemoglobin WBC White blood count

HbA1c Glycosylated hemoglobin X-ray Roentgenogram

Hct Hematocrit

HIV Human immunodeficiency virus